Vajiram CA Test-10 (2024)

You might also like

Download as pdf or txt
Download as pdf or txt
You are on page 1of 74

.

DO NOT OPEN THIS BOOKLET UNTIL YOU ARE ASKED TO DO SO

VAJIRAM & RAVI


PowerUp Current Affairs Test Series - 2024
CA Test – 10 – (CA3320)

(Time Allowed: 2 hours) (Maximum Marks: 200)

INSTRUCTIONS
1. Immediately after the commencement of the examination, you should check
that this test booklet does not have any unprinted or torn or missing pages or
items etc. If so, get it replaced by a complete test booklet.
2. You have to enter your Roll Number on the Test
Booklet in the Box provided alongside. DO NOT
write anything else on the Test Booklet.
3. This Test Booklet contains 100 items (questions). Each item comprises four
responses (answers). You will select the response which you want to mark on
the Answer Sheet.
In case you feel that there is more than one correct response, mark the response
which you consider the best. In any case, choose ONLY ONE response for each
item.
4. You have to mark all your responses ONLY on the separate Answer Sheet
provided. See directions in the Answer Sheet. All items carry equal marks.
5. Before you proceed to mark in the Answer Sheet the response to various items
in the Test Booklet, you have to fill in some particulars in the Answer Sheet.
6. There will be penalty for wrong answers marked by a candidate. For each wrong
answer, one-third of the marks assigned to that question will be deducted as
penalty.
7. If a candidate gives more than one answer, it will be treated as a wrong answer,
even if one of the given answers happens to be correct.
8. If a question is left blank, i.e., no answer is given by the candidate, there will be
no penalty for that question.
9. After you have completed filling in all your responses on the Answer Sheet and
the examination has concluded, you should hand over to the Invigilator only the
Answer Sheet. You are permitted to take away with you the Test Booklet.

DO NOT OPEN THIS BOOKLET UNTIL YOU ARE ASKED TO DO SO


VAJIRAM & RAVI

.
1. In India, which one of the following is 4. Which one of the following is the primary
responsible for the release of the all India objective of the POEM platform of the Indian
Consumer Price Index (CPI)? Space Research Organisation (ISRO)?
(a) Reserve Bank of India (a) To allow users to view geo-spatial and
(b) Department of Economic Affairs, Ministry thematic maps
of Finance (b) To capture cosmic sound from other solar
(c) National Statistical Office (NSO) systems
(d) Office of the Economic Advisor, Ministry (c) To support the development of new
propulsion technologies for space
of Commerce and Industry
missions
(d) To perform in-orbit experiments using the
2. Consider the following statements with
discarded stage of workhorse rocket
reference to Virtual Digital Assets (VDAs)
service providers:
5. Consider the following statements with
1. The Prevention of Money Laundering Act
reference to Pangolin:
(PMLA), 2002 covers only those VDA 1. It is the only scaly mammal found on
service providers with a physical presence Earth.
in India. 2. Indian Pangolin is the only pangolin
2. All VDA Service Providers operating in species found in India.
India are required to be registered with 3. The colour of the scales of the Indian
the Financial Intelligence Unit (FIU) under Pangolin varies as per its surroundings.
the Union Ministry of Finance. How many of the above statements are
Which of the statements given above is/are correct?
correct? (a) Only one
(a) 1 only (b) Only two
(b) 2 only (c) All three
(c) Both 1 and 2 (d) None
(d) Neither 1 nor 2
6. Consider the following statements with
3. Consider the following pairs: reference to the Agreement on the Prohibition
of Attack against Nuclear Installations and
Sl. Geographical Associated State
No. Indication (GI) Facilities:
Tag 1. It mandated India and Pakistan to inform
1. Kapdaganda : Odisha each other of nuclear installations on an
Shawl
annual basis.
2. Wancho : Sikkim
Wooden Craft 2. As per the agreement, nuclear facilities
3. Kalonunia : West Bengal include any facility with irradiated nuclear
4. Kachchhi : Gujarat fuel and materials.
Kharek Which of the statements given above is/are
Which of the pairs given above are correctly correct?
matched? (a) 1 only
(a) 1 and 2 only (b) 2 only
(b) 3 and 4 only (c) Both 1 and 2
(c) 1, 2 and 3 only (d) Neither 1 nor 2
(d) 1, 3 and 4 only
Vajiram & Ravi PowerUp Current Affairs Test Series (2024) 1
Test No. – 10 (CA3320)
VAJIRAM & RAVI

.
7. It is the only dedicated rehabilitation center 10. Consider the following statements with
for the Indian Star Tortoise in the country. reference to the Central Armed Police Forces
It is located in the rain shadow region of (CAPFs):
the Western Ghats. It is covered by the 1. The Border Security Force (BSF) and
Eravikulam National Park on the south. This Sashastra Seema Bal (SSB) are the only
is the most likely description of:
border guarding forces under CAPFs.
(a) Bhadra Wildlife Sanctuary
2. The Central Reserve Police Forces
(b) Chinnar Wildlife Sanctuary
(c) Mudumalai Wildlife Sanctuary (CRPF) is deployed in matters relating to
(d) Radhanagari Wildlife Sanctuary internal security and maintenance of law
and order.
8. Consider the following statements with 3. Assam Rifles function under the
reference to the Unlawful Activities operational control of the Union Ministry
(Prevention) Amendment Act (UAPA), 2019: of Defence.
1. It clearly defines the words “terror” and How many of the above statements are
“terrorist”. correct?
2. The government is required to give the (a) Only one
individual an opportunity to be heard (b) Only two
before designating him/her as terrorist.
(c) All three
3. The review committee set up under the
(d) None
Act can order the government to delete the
name of the individual from the schedule
that lists “terrorists.” 11. Consider the following statements with
How many of the above statements are reference to the Convention on the Prevention
correct? and Punishment of the Crime of Genocide,
(a) Only one 1948:
(b) Only two 1. Imposing measures to prevent birth in a
(c) All three group can be termed as genocide.
(d) None 2. It does not mandate any punishment
to private individuals for the crime of
9. Consider the following statements:
genocide.
1. A savings as well as a current account
3. Genocide can be considered a political
can be termed as inoperative if there are
no transactions in the account for over a crime for extradition purposes under the
period of one year. Convention.
2. Both customer-induced transactions How many of the above statements are
and bank induced transactions shall be correct?
considered before declaring an account (a) Only one
inoperative. (b) Only two
3. Zero balance accounts cannot be (c) All three
declared as inoperative even if there are (d) None
no transactions for a period of more than
two years. 12. The Green Cover Index aims to measure
How many of the above statements are
green cover at pan India level for:
correct?
(a) Coastal Regulation Zones
(a) Only one
(b) Only two (b) Buffer Zones of Protected Areas
(c) All three (c) National Highways
(d) None (d) Special Economic Zones

2 Vajiram & Ravi PowerUp Current Affairs Test Series (2024)


Test No. – 10 (CA3320)
VAJIRAM & RAVI

.
13. The National Transit Pass System (NTPS) 16. The ‘Acceptance of Necessity Accord’ was
was recently launched by the Government of recently seen in news in the context of:
India to: (a) humanitarian aid in conflict zones
(a) develop a secure national database (b) disaster management
for tracking the movement of public (c) defence procurement and acquisition
transportation vehicles (d) environmental clearance for construction
(b) facilitate movement for smooth organ projects
transplantation
(c) create a national database of all 17. Consider the following statements with
unorganised workers including gig and reference to the Kalaram temple of Nashik,
platform workers seeded with Aadhaar Maharashtra:
(d) enable seamless transit of timber across 1. It was built by Narasimhavarman I of the
the nation Pallava dynasty.
2. The temple consists of black statues of
14. Which of the following are the tools of
Lord Ram and Hanuman.
Recombinant DNA Technology?
3. Dr B.R. Ambedkar initiated a temple entry
1. Restriction enzymes
satyagraha at this temple in 1930.
2. Selectable markers
How many of the above statements are
3. Cytokine barriers
correct?
4. Vectors
(a) Only one
Select the correct answer using the code
(b) Only two
given below:
(c) All three
(a) 2 and 3 only
(d) None
(b) 1 and 4 only
(c) 1, 2 and 4 only
(d) 2, 3 and 4 only 18. Consider the following statements with
reference to plasma waves:
15. Consider the following statements with 1. They are short-time scale fluctuations in
reference to rock glacier: electric and magnetic field observations.
1. It is a mass of mud and water that moves 2. They can act as a cleaning agent for the
slowly down a mountain under the Earth’s radiation belt.
influence of gravity. 3. They occur only in high frequencies.
2. It forms from the debris-covered ice when 4. They are often observed in the Earth’s
a glacier recedes or thaws. magnetosphere.
Which of the statements given above is/are How many of the above statements are
correct? correct?
(a) 1 only (a) Only one
(b) 2 only (b) Only two
(c) Both 1 and 2 (c) Only three
(d) Neither 1 nor 2 (d) All four

Vajiram & Ravi PowerUp Current Affairs Test Series (2024) 3


Test No. – 10 (CA3320)
VAJIRAM & RAVI

.
19. With reference to Supreme Court Legal 22. Which one of the following is the primary
Services Committee, consider the following function of the thylakoid membrane in plant
statements: cells?
1. It primarily aims to help the Supreme
(a) Storage of nutrients
Court in clearing pending cases related to
(b) Cellular respiration
infringement of Fundamental Rights.
(c) Photosynthesis
2. It is constituted by the President of India
in consultation with the Supreme Court (d) Synthesis of protein
collegium.
3. The Chief Justice of India is the ex-officio 23. Which of the following are the sub-schemes
chairman of the committee. under the PRITHvi VIgyan (PRITHVI)
How many of the above statements are Scheme?
correct?
1. Atmosphere & Climate Research-
(a) Only one
Modelling Observing Systems & Services
(b) Only two
(c) All three (ACROSS)
(d) None 2. Ocean Services, Modelling Application,
Resources and Technology (O-SMART)
20. The term ‘Zosurabalpin’, recently seen in 3. Polar Science and Cryosphere Research
news, is a: (PACER)
(a) Neglected Tropical Disease (NTD)
4. Seismology and Geosciences (SAGE)
(b) new class of antibiotics to tackle a drug-
5. Synergistic Training program Utilising the
resistant bacterium
Scientific and Technological Infrastructure
(c) pesticide to combat wheat blast
(d) newly discovered subatomic particle (STUTI)
associated with dark matter Select the correct answer using the code
given below:
21. With reference to Sisal plant, consider the (a) 1 and 3 only
following statements: (b) 2, 4 and 5 only
1. It is a semi-perennial fiber-producing
(c) 1, 2, 3 and 4 only
plant that produces only one flower in its
(d) 1, 2, 3, 4 and 5
lifetime.
2. It is drought-resistant and grows well in
arid and semi-arid regions. 24. The Congo River, the second-longest and
3. It can be used to make paper, clothes and world's deepest recorded river in Africa,
ropes. drains into:
How many of the above statements are (a) Indian Ocean
correct? (b) Mediterranean Sea
(a) Only one
(c) Atlantic Ocean
(b) Only two
(d) Red Sea
(c) All three
(d) None

4 Vajiram & Ravi PowerUp Current Affairs Test Series (2024)


Test No. – 10 (CA3320)
VAJIRAM & RAVI

.
25. Consider the following statements with 28. The European Space Agency has recently
reference to the Gulf of Aden: announced Proba-3 Mission with an aim to:
1. It is situated between the coasts of Arabia (a) study the geological composition of
and the Horn of Africa. asteroids in asteroid belt
2. It is connected to the Red Sea by the (b) measure the gravitational waves from
Strait of Bab el Mandeb. merging black holes and neutron stars
3. It is bordered by Egypt, Jordan and (c) map the surface and subsurface of Mars
Sudan. using radar and optical sensors
Which of the statements given above are (d) observe the Sun’s corona in the visible
wavelength range
correct?
(a) 1 and 2 only
29. With reference to the Telecommunications
(b) 2 and 3 only
Act, 2023, consider the following statements:
(c) 1 and 3 only
1. It creates a non-lapsable fund for the
(d) 1, 2 and 3
development of telecommunication
infrastructure.
26. In which one of the following protected areas
2. It empowers State Governments to
of India is the critically endangered Bugun
provide authorisation for possessing radio
Liocichla bird exclusively found? equipment.
(a) Eaglenest Wildlife Sanctuary 3. It allows the Central Government to
(b) Jim Corbett National Park reclaim the unutilised spectrum.
(c) Pobitora Wildlife Sanctuary 4. Over-the-top (OTT) services are excluded
(d) Namdapha National Park from the purview of the Act.
How many of the above statements are
27. Consider the following statements with correct?
reference to the Periodic Labour Force (a) Only one
Survey (PLFS) 2022-23: (b) Only two
1. Unlike males, the Working Population (c) Only three
Ratio (WPR) has increased for females in (d) All four
both rural and urban areas.
2. The Unemployment Rate (UR) in the age 30. Which one of the following is the primary
group 15-29 is twice that across all age objective of Mulya Pravah 2.0, a guideline
groups. issued by the University Grants Commission
3. The increase in employment of females (UGC) in India for higher education
is majorly concentrated in the agriculture institutions?
(a) Regulating the recruitment of visiting
sector and self-employment category.
faculty in Central universities
Which of the statements given above is/are
(b) Enhancing research and innovation
correct?
capacity in higher education institutions
(a) 2 only
(c) Allowing foreign higher education
(b) 3 only
institutions to set up campuses in India
(c) 1 and 3 only
(d) Instilling human values and professional
(d) 1, 2 and 3
ethics in higher education institutions
Vajiram & Ravi PowerUp Current Affairs Test Series (2024) 5
Test No. – 10 (CA3320)
VAJIRAM & RAVI

.
31. Consider the following statements: 34. Consider the following statements with
1. The United Arab Emirates (UAE) is the reference to magnetic cooling:
third largest trading partner of India. 1. It is the reversible temperature change of
2. UAE is the largest export destination for a magnetic material when it is subjected
India. to an externally applied magnetic field.
3. India and UAE have signed a 2. Unlike the vapour-cycle refrigeration
Comprehensive Economic Partnership technology, it is an energy-efficient and
Agreement (CEPA). environment-friendly cooling technology.
4. UAE is a full time member of the BRICS Which of the statements given above is/are
bloc. correct?
Which of the statements given above are (a) 1 only
correct? (b) 2 only
(a) 1 and 3 only (c) Both 1 and 2
(b) 2 and 4 only (d) Neither 1 nor 2
(c) 1, 3 and 4 only
(d) 2, 3 and 4 only 35. NUTEC Plastics initiative to address the
global challenge of plastic pollution is recently
32. Protection and Indemnity (P&I) Club, recently launched by which one of the following
seen in news, is related to: organisations?
(a) protection of climate related investments (a) World Wide Fund for Nature
(b) insurance cover for shipowners for (b) International Atomic Energy Agency
maritime risks (c) United Nations Environment Programme
(c) monitoring of human rights violations in (d) World Economic Forum
conflict zones
(d) protection of Intellectual Property Rights 36. Recently, a deal was made between the Union
(IPRs) Government and multiple States and Union
Territories (UT) related to the Republic Day
33. Consider the following pairs: parade tableaux selection. In this context,
consider the following statements:
Sl. Missile Feature
No. 1. The presentation of tableaux is done only
1. Rudram : Quasi-ballistic surface- by the States and UTs.
to-surface missile 2. Every State and UT are allowed to
2. Akash NG : Medium range surface-
to-air missile showcase their tableaux on each Republic
3. Nirbhay : Subsonic cruise missile Day.
4. Pralay : Anti-radiation missile 3. The Union Ministry of Defence is
responsible for the selection of tableaux.
How many of the above pairs are correctly How many of the above statements are
matched? correct?
(a) Only one pair (a) Only one
(b) Only two pairs (b) Only two
(c) Only three pairs (c) All three
(d) All four pairs (d) None

6 Vajiram & Ravi PowerUp Current Affairs Test Series (2024)


Test No. – 10 (CA3320)
VAJIRAM & RAVI

.
37. The term 'Sidersaura Marae', sometimes 40. With reference to Cold Dark Matter (CDM),
mentioned in the news, is a/an: consider the following statements:
(a) dinosaur fossil 1. It interacts very weakly with ordinary
matter.
(b) rare butterfly species native to Western
2. It moves slowly compared to the speed of
Ghats
light.
(c) bacteria used for bioremediation 3. It is associated with the growth and
(d) invasive species of aquatic algae evolution of galaxies.
How many of the above statements are
38. Consider the following statements with correct?
reference to Einstein Probe: (a) Only one
(b) Only two
1. It is launched by the National Aeronautics
(c) All three
and Space Administration (NASA).
(d) None
2. It is aimed at capturing the first light arising
out of a supernova explosion. 41. Drishti 10 Starliner, recently seen in the news,
Which of the statements given above is/are is a/an:
correct? (a) advanced remote sensing satellite
(a) 1 only (b) medium-altitude long-endurance drone

(b) 2 only for the Indian Navy


(c) reusable space capsule for human
(c) Both 1 and 2
spaceflight
(d) Neither 1 nor 2
(d) supercomputer for weather forecasting
and climate research
39. Which one of the following statements best
describes the term ‘Meissner Effect’? 42. With reference to ‘quantum gates’, consider
(a) It is the effect generated from the transfer the following statements:
of electrons between atoms and molecules 1. They are basic building blocks of quantum
circuits.
involved in a chemical reaction.
2. They can manipulate the quantum state
(b) It refers to the self-cleaning properties
of qubits through entanglement.
of leaves of certain flowers as a result of
3. Unlike classical gates, they preserve the
ultra hydrophobicity. information and do not result in a loss of
(c) It refers to the expulsion of the magnetic data.
field from the interior of a material during How many of the above statements are
its transition to a superconductor. correct?

(d) It is the decrease in energy of an X-ray or (a) Only one


(b) Only two
gamma ray photon, when it interacts with
(c) All three
matter.
(d) None

Vajiram & Ravi PowerUp Current Affairs Test Series (2024) 7


Test No. – 10 (CA3320)
VAJIRAM & RAVI

.
43. Green Fuels Alliance India, recently seen in 46. Consider the following:
news, is a: 1. Brick industry
(a) multinational effort led by India to address
2. Aluminium smelters
climate change through renewable energy
3. Coal burning
(b) joint venture of Indian and French
companies to develop and market biofuels 4. Phosphate fertilisers
for aviation and maritime sectors How many of the above are the sources of
(c) strategic initiative between India and fluoride contamination in groundwater?
Denmark to collaborate in the sustainable (a) Only one
energy sector and to achieve carbon
(b) Only two
neutrality
(d) research consortium of Indian and (c) Only three
German universities and institutes to (d) All four
explore new technologies for green fuels
47. With reference to cosmic rays, consider the
44. In the context of evolution of Earth, ‘Great
following statements:
Oxidation Event’ is related to:
1. They are high-energy particles primarily
(a) decrease in the level of carbon dioxide
due to its consumption by photosynthetic originating from supernovae and pulsars.
organisms 2. They consist of charged particles including
(b) formation of the ozone layer in the protons and heavier atomic nuclei.
atmosphere 3. Only low-intensity cosmic rays reach the
(c) increase in methane concentration due
surface of the Earth.
to increase in decomposition of organic
matter How many of the above statements are
(d) increase in concentration of oxygen in the correct?
atmosphere and oceans (a) Only one
(b) Only two
45. With reference to the Lentil (Masoor) crop,
(c) All three
consider the following statements:
(d) None
1. It requires a high temperature during its
vegetative growth.
2. India mainly relies on import of lentils to 48. Tricholime, a newly developed granular lime-
meet its domestic demand. based formulation, is primarily used for:
3. The Government of India has provided (a) stimulating the production of nitrogen-
the highest increase in Minimum Support
fixing enzymes
Price (MSP) for lentils among all rabi
crops in 2023. (b) preventing the leaching of phosphates
How many of the above statements are from the soil profile
correct? (c) neutralisation of soil acidity
(a) Only one (d) enhancing water retention in arid soils
(b) Only two
(c) All three
(d) None
8 Vajiram & Ravi PowerUp Current Affairs Test Series (2024)
Test No. – 10 (CA3320)
VAJIRAM & RAVI

.
49. Consider the following statements with 52. The National Essential Diagnostics List
reference to X-Rays: (NEDL) which contains the minimum
1. X-Rays are a form of electromagnetic diagnostic tests that should be available at
radiation which can pass through the healthcare facilities is maintained by:
human body. (a) Indian Council of Medical Research
2. Polarised X-Rays can help in measuring (b) Central Drug Standard Control
the direction of magnetic fields.
Organization
3. Earth's surface reflects most of the X-Rays
(c) Department of Pharmaceuticals
to outer space.
(d) National Medical Council
Which of the statements given above are
correct?
(a) 1 and 2 only 53. Consider the following statements:
(b) 2 and 3 only Statement-I: An electric current in a
(c) 1 and 3 only superconductor can persist indefinitely.
(d) 1, 2 and 3 Statement-II: A superconductor has no
electrical resistance.
50. Consider the following statements with Which one of the following is correct in respect
reference to the Himalayan Wolf: of the above statements?
1. It is endemic to the Northern States of (a) Both Statement-I and Statement-II are
India. correct and Statement-II is the correct
2. It is listed as Vulnerable in the International explanation for Statement I
Union for Conservation of Nature (IUCN)’s
(b) Both statement-I i and Statement-II are
Red List of Threatened Species.
correct but Statement-II is not the correct
3. Hybridization with dogs is an emerging
explanation for Statement-I
threat to the species.
(c) Statement-I is correct but Statement-II is
Which of the statements given above are
correct? incorrect
(a) 1 and 2 only (d) Statement-I is incorrect but Statement-II
(b) 2 and 3 only is correct
(c) 1 and 3 only
(d) 1, 2 and 3 54. With reference to ‘Crystal Awards’, consider
the following statements:
51. With reference to Light Emitting Diodes 1. They are announced annually by the
(LEDs), consider the following statements: United Nations Educational, Scientific
1. They work on the principle of and Cultural Organization (UNESCO).
electroluminescence. 2. They celebrate the achievements of
2. They can produce all three primary leading artists who are role models for all
colours - red, green and blue.
leaders of society.
3. Unlike Compact Fluorescent Lighting
3. No Indian citizen has received this award
(CFL), the LEDs emit light in a specific
till now.
direction.
How many of the statements given above are How many of the statements given above are
correct? correct?
(a) Only one (a) Only one
(b) Only two (b) Only two
(c) All three (c) All three
(d) None (d) None
Vajiram & Ravi PowerUp Current Affairs Test Series (2024) 9
Test No. – 10 (CA3320)
VAJIRAM & RAVI

.
55. With reference to India’s biodiversity, 59. With reference to the International Court of
‘Punganur, Sanchori and Masilum’ are the Justice, consider the following statements:
breeds of: 1. It is the highest judicial body of the United
(a) cows Nations.
(b) goats 2. It has no jurisdiction to deal with
(c) sheep applications from private entities or non-
(d) camels governmental organisations.
3. It has no jurisdiction to try individuals
56. Consider the following statements with accused of war crimes or crimes against
reference to the Vaccine Safety Net: humanity.
1. It is a global network of websites that How many of the above statements are
provides reliable information on vaccine correct?
safety. (a) Only one
2. It is established by the World Health (b) Only two
Organization (WHO). (c) All three
Which of the statements given above is/are (d) None
correct?
(a) 1 only 60. Consider the following statements:
(b) 2 only 1. Thorium is more abundant in the Earth's
(c) Both 1 and 2 crust than uranium.
(d) Neither 1 nor 2 2. Thorium can undergo nuclear reactions to
form uranium-233.
57. Which of the following satellites are used 3. India has the largest reserves of monazite
by India in weather monitoring and warning and ilmenite in the world.
services? How many of the above statements are
1. Kalpana 1 correct?
2. INSAT 3D (a) Only one
3. CARTOSAT -3 (b) Only two
4. SCATSAT-1 (c) All three
5. GSAT 7B (d) None
Select the correct answer using the code
given below:
61. With reference to the ‘self-growing robot’,
(a) 1 and 2 only
consider the following statements:
(b) 1, 2 and 4 only
1. It responds to gravity and light like the
(c) 3, 4 and 5 only
adaptive behaviour of plants.
(d) 1, 2, 3, 4 and 5
2. It can autonomously construct 3-D
structures through additive manufacturing
58. What are ‘Ice Nucleators’?
techniques.
(a) Coolants used in nuclear reactors to
3. It cannot penetrate dense media like soil.
remove heat
How many of the above statements are
(b) Substances used to liquefy carbon dioxide
correct?
for the production of dry ice
(a) Only one
(c) Substances that facilitate the transition of
(b) Only two
water from liquid state to ice
(c) All three
(d) Chemicals used to maintain temperature
(d) None
in cryogenic storage tanks

10 Vajiram & Ravi PowerUp Current Affairs Test Series (2024)


Test No. – 10 (CA3320)
VAJIRAM & RAVI

.
62. Which one of the following separates 66. Consider the following statements with
Lakshadweep and Maldives? reference to seamounts:
(a) Eight degree channel 1. They are formed through volcanic activity.
(b) Nine degree channel 2. They can cause localised ocean upwelling.
(c) Ten degree channel 3. Tamu Massif in the Pacific Ocean is the
(d) Eleven degree channel largest seamount in the world.
How many of the above statements are
63. With reference to ‘Soda lakes’, consider the correct?
(a) Only one
following statements:
(b) Only two
1. They are generally alkaline and have a
(c) All three
higher pH value.
(d) None
2. They are characterised by a rich diversity
of microbial life.
67. With reference to the Design Linked Incentive
3. They are mostly found within endorheic (DLI) scheme, consider the following
(closed) basins. statements:
How many of the above statements are 1. It aims to strengthen the semiconductor
correct? chip design ecosystem in the country.
(a) Only one 2. Companies should retain their domestic
(b) Only two status at least for three years after the
(c) All three approval to claim financial incentives
(d) None under the scheme.
3. The Department of Science and
64. The Sulfur Fluoride Exchange (SuFEx) Technology acts as the nodal agency for
chemistry, sometimes seen in the news, is its implementation.
mostly used in: How many of the above statements are
(a) therapeutic drugs correct?
(b) industrial solvents (a) Only one
(c) artificial sweeteners (b) Only two
(c) All three
(d) oil extraction
(d) None

65. With reference to the Raptors, consider the


68. With reference to ‘3-D Organoids’, consider
following statements:
the following statements:
1. They act as both apex predators as well
1. They are miniaturised versions of tissues
as scavengers in the food chain. derived from cells with stem potential.
2. They lay their eggs in the nests of other 2. They can only be grown out of embryonic
birds. stem cells.
3. They are restricted only to mountainous 3. They can help to understand colour
habitats. blindness and age-related vision loss.
How many of the above statements are How many of the above statements are
correct? correct?
(a) Only one (a) Only one
(b) Only two (b) Only two
(c) All three (c) All three
(d) None (d) None

Vajiram & Ravi PowerUp Current Affairs Test Series (2024) 11


Test No. – 10 (CA3320)
VAJIRAM & RAVI

.
69. Consider the following statements: 72. The Global Goal on Adaptation (GGA) to
1. While probiotics are live bacteria found increase funding towards countries’ climate
in foods, prebiotics are food for these change adaptation, was first established
bacteria. under which one of the following frameworks?
(a) Paris Agreement
2. Synbiotic foods contain both beneficial
(b) Montreal Protocol
bacteria and a prebiotic source of fiber.
(c) Kunming Declaration
Which of the statements given above is/are (d) London Convention
correct?
(a) 1 only 73. Consider the following statements with
(b) 2 only reference to the Indo-Pacific Triangular
(c) Both 1 and 2 Development Cooperation Fund:
(d) Neither 1 nor 2 1. It is a joint initiative of India and France.
2. It aims to support innovations to scale
up green technologies in the Indo-Pacific
70. Consider the following statements :
region.
1. In end-to-end encryption, the message is
Which of the statements given above is/are
encrypted only in transit and not at rest.
correct?
2. In asymmetric encryption, the same (a) 1 only
key is used to encrypt and decrypt the (b) 2 only
information. (c) Both 1 and 2
3. WiFi passwords are encrypted through a (d) Neither 1 nor 2
symmetric encryption algorithm.
How many of the above statements are 74. Which one of the following is the primary
correct? objective of NASA's Lunar Reconnaissance
Orbiter (LRO)?
(a) Only one
(a) Identify sites favourable for safe future
(b) Only two
robotic and human lunar missions
(c) All three (b) Study asteroids around the Moon
(d) None (c) Collect on-site samples of lunar rocks
(d) Measure the seismicity around the south
71. With reference to the Indian economy, which pole of the moon
of the following statements is/are correct?
1. At present, direct taxes constitute more 75. Consider the following statements:
than half of the government’s total tax 1. Unlike venture capital firms which invest
in startups, private equity firms buy
revenue.
already established companies with low/
2. As the economy grows and per capita
zero profits.
income increases, the tax-to-GDP ratio
2. Unlike private equity firms, venture capital
also increases. firms deal with both cash and debt.
Select the correct answer using the code Which of the statements given above is/are
given below: correct?
(a) 1 only (a) 1 only
(b) 2 only (b) 2 only
(c) Both 1 and 2 (c) Both 1 and 2
(d) Neither 1 nor 2 (d) Neither 1 nor 2

12 Vajiram & Ravi PowerUp Current Affairs Test Series (2024)


Test No. – 10 (CA3320)
VAJIRAM & RAVI

.
76. Consider the following statements with 79. With reference to the UNESCO World Heritage
reference to Trust and Society: Sites, consider the following statements:
1. Both are governed by the Societies 1. India has the largest number of UNESCO
Registration Act of 1860. World Heritage sites in Asia.
2. A minimum of ten members are required 2. Gujarat has the highest number of
for the formation of both. UNESCO World Heritage Sites in India.
3. Unlike a Society, a Trust has an elected 3. Khangchendzonga National Park is the
body to manage its affairs. only Mixed Heritage Site in India.
How many of the above statements are How many of the above statements are
correct? correct?
(a) Only one (a) Only one
(b) Only two (b) Only two
(c) All three
(c) All three
(d) None
(d) None

80. With reference to e-vehicles, what is “Range


77. Consider the following statements with
anxiety”?
reference to the Ice Memory initiative:
(a) Insufficient charging infrastructure for
1. It primarily aims to conserve the glaciers
e-vehicles
which serve as the source of Himalayan
(b) Time-consuming recharge of e-vehicle
rivers.
batteries to travel long distances
2. It will analyse reference ice cores to create
(c) Higher cost of e-vehicles as a significant
an open-access database.
barrier to wider adoption
Which of the statements given above is/are
(d) Lack of sustainable disposal methods for
correct?
e-vehicle batteries
(a) 1 only
(b) 2 only 81. With reference to ‘Green roofs’, consider the
(c) Both 1 and 2 following statements:
(d) Neither 1 nor 2 1. They can reduce stormwater runoff in the
urban environment.
78. The term ‘multi-omics’, sometimes seen in 2. They are effective in filtering the pollutants
the news, is related to: from the rainwater.
(a) study of weather patterns and their impact 3. They can reduce city-wide ambient
on agriculture temperatures.
(b) treatment of applications of artificial How many of the above statements are
intelligence in genetic diseases correct?
(c) conservation of endangered species in an (a) Only one
ecosystem (b) Only two
(d) integration of genomics with studies on (c) All three
proteins and metabolites in the body (d) None

Vajiram & Ravi PowerUp Current Affairs Test Series (2024) 13


Test No. – 10 (CA3320)
VAJIRAM & RAVI

.
82. ‘eROSITA’, often mentioned in the news, is a: 86. Consider the following:
(a) spacecraft to analyse the outer galaxies 1. Oil seeds
(b) space mission to test plant-microbe
2. Wastewater treatment sludge
interactions in space
(c) X-ray telescope to deliver sharp images 3. Algae
over very large areas of the sky 4. Wood mill waste
(d) satellite to measure X-ray polarisation
5. Manures
from celestial sources
How many of the above can be used for
83. Consider the following pairs: making Sustainable Aviation Fuel?
Sl. Ramsar site State (a) Only two
No.
1. Sirpur wetland : Madhya Pradesh (b) Only three
2. Longwood Shola : Karnataka (c) Only four
Reserve Forest
(d) All five
3. Kanwar lake : Himachal Pradesh
4. Karaivetti Bird : Tamil Nadu
Sanctuary 87. Which one of the following statements best
How many of the above pairs are correctly describes the term ‘Loud Budgeting’?
matched?
(a) A financial strategy characterised by the
(a) Only one pair
(b) Only two pairs vocal and intentional allocation of funds
(c) Only three pairs towards personal aspirations
(d) All four pairs (b) A budgeting mechanism which involves

84. The textiles known as Apatani, Monpa and Tai announcement of the borrowing
Khamti were recently granted Geographical requirements for the financial year
Indication (GI) tags. They belong to which (c) A budgeting mechanism where the targets
one of the following States?
of the economy in terms of quantity are
(a) Arunachal Pradesh
(b) Sikkim explicitly mentioned
(c) Tripura (d) A financial strategy to increase the
(d) Odisha
investments in public infrastructure in an
85. With reference to the Asia-Africa Growth economy
Corridor (AAGC), consider the following
statements: 88. In the context of banking, the ‘CASA ratio’
1. It is launched under the aegis of the G7
group. reflects the bank’s capacity to:
2. It aims to establish a Pan-Africa (a) invest in international markets
e-Network for capacity building and skill (b) long term financing of infrastructure
development.
projects
Which of the statements given above is/are
correct? (c) raise money with lower borrowing costs
(a) 1 only (d) withstand a transient rise in Non-
(b) 2 only
Performing Assets
(c) Both 1 and 2
(d) Neither 1 nor 2

14 Vajiram & Ravi PowerUp Current Affairs Test Series (2024)


Test No. – 10 (CA3320)
VAJIRAM & RAVI

.
89. With reference to the International 92. The ‘BioRescue Project’, recently seen in the
Classification of Diseases-11 (ICD-11), news, aims to:
consider the following statements:
(a) increase the surveillance of cheetah
1. It was adopted under the aegis of the
World Health Assembly to collect data on habitat
communicable and non-communicable (b) surveillance of endangered species to
mortality statistics. reduce trafficking
2. Ayurveda, Unani and Siddha (ASU)
(c) track the movement of elephants
systems of medicines have the same
morbidity codes across the globe under (d) protect Northern White Rhino from
ICD-11. extinction
Which of the statements given above is/are
correct?
(a) 1 only 93. Consider the following statements:
(b) 2 only 1. Deep learning uses neural networks to do
(c) Both 1 and 2 complex machine learning tasks without
(d) Neither 1 nor 2
any human intervention.

90. Consider the following: 2. Unlike machine learning, deep learning


1. Diversion of public funds eliminates the data pre-processing to
2. Tax fraud process unstructured data.
3. Nepotistic appointments in the civil service
Which of the statements given above is/are
4. Illicit financial flows
5. Money-laundering correct?
How many of the above are considered as (a) 1 only
parameters of corruption under the Corruption (b) 2 only
Perception Index released by Transparency
(c) Both 1 and 2
International?
(a) Only two (d) Neither 1 nor 2
(b) Only three
(c) Only four 94. Consider the following statements with
(d) All five
reference to the Altermagnets:
91. With reference to ‘E Ink display’, consider the 1. They have both ferromagnetic and
following statements: antiferromagnetic properties.
1. It uses charged particles in the fluid inside
2. They exhibit a unique feature of spin
the display.
2. It does not reflect the light and uses a polarization which allows electron spins
backlight to function. to align in a particular direction.
3. It has a slow refresh rate compared to Which of the statements given above is/are
Liquid-Crystal Displays (LCD).
correct?
How many of the statements given above are
correct? (a) 1 only
(a) Only one (b) 2 only
(b) Only two (c) Both 1 and 2
(c) All three
(d) Neither 1 nor 2
(d) None
Vajiram & Ravi PowerUp Current Affairs Test Series (2024) 15
Test No. – 10 (CA3320)
VAJIRAM & RAVI

.
95. With reference to the Codex Alimentarius 98. Consider the following activities:
Commission, consider the following 1. Personalised breathing exercises
statements:
2. Contactless payment
1. It was jointly established by the Food and
3. Control home appliances
Agriculture Organisation and the World
Health Organisation. 4. Biometric authentication
2. It contributes to the harmonization of food At the present level of technology, how many
standards across the globe. of the above can be carried out by Smart
3. Its standards are recognized by the World
Rings?
Trade Organisation for the resolution of
trade disputes. (a) Only one
How many of the above statements are (b) Only two
correct? (c) Only three
(a) Only one (d) All four
(b) Only two
(c) All three
(d) None 99. The term ‘Cyclocenes’ is sometimes seen in
the news in the context of:
96. Which one of the following statements best (a) disaster management
describes the term ‘Metamaterials’?
(b) biotechnology
(a) Materials whose electrical conductivity
(c) organometallic chemistry
falls in between that of conductors and
insulators (d) geoengineering
(b) Non-metallic materials that are typically
formed by heating and molding clay 100. Consider the following statements with
(c) Polymers that can restore their original
reference to the election candidates in India:
shape after being stretched to a large
extent 1. A maximum of two nomination papers can
(d) Materials manufactured artificially with be filed by a candidate or on his behalf for
unique nanoscale patterns or geometry a constituency.
2. A candidate is required to make a separate
97. Consider the following species in India:
security deposit for each of the nomination
1. Dattatreya Night Frog
2. Malabar large-spotted civet papers filed from the same constituency.
3. Forest owlet 3. Before filing a nomination, the candidate
4. Namdapha flying squirrel is required to mention a choice of election
5. Sociable lapwing
symbols.
How many of the above are classified as
How many of the above statements are
critically endangered in the IUCN Red List of
Threatened Species? correct?
(a) Only two (a) Only one
(b) Only three (b) Only two
(c) Only four
(c) All three
(d) All five
(d) None

16 Vajiram & Ravi PowerUp Current Affairs Test Series (2024)


Test No. – 10 (CA3320)
.
VAJIRAM & RAVI
Institute for IAS Examination

Current Affairs Test – 10


(CA3320)

Answer Key
&
Detailed Answer Explanation
.
.
PowerUp Current Affairs Test – 10 (CA3320) - Answer Key
1. (c) 11. (a) 21. (c) 31. (c) 41. (b) 51. (c) 61. (b) 71. (c) 81. (c) 91. (b)
2. (b) 12. (c) 22. (c) 32. (b) 42. (c) 52. (a) 62. (a) 72. (a) 82. (c) 92. (d)
3. (d) 13. (d) 23. (c) 33. (b) 43. (c) 53. (a) 63. (c) 73. (c) 83. (b) 93. (c)
4. (d) 14. (c) 24. (c) 34. (c) 44. (d) 54. (a) 64. (a) 74. (a) 84. (a) 94. (c)
5. (b) 15. (c) 25. (a) 35. (b) 45. (b) 55. (a) 65. (a) 75. (a) 85. (b) 95. (c)
6. (c) 16. (c) 26. (a) 36. (a) 46. (d) 56. (c) 66. (b) 76. (d) 86. (d) 96. (d)
7. (b) 17. (b) 27. (c) 37. (a) 47. (c) 57. (b) 67. (b) 77. (b) 87. (a) 97. (c)
8. (a) 18. (c) 28. (d) 38. (b) 48. (c) 58. (c) 68. (b) 78. (d) 88. (c) 98. (d)
9. (a) 19. (d) 29. (c) 39. (c) 49. (a) 59. (c) 69. (c) 79. (a) 89. (c) 99. (c)
10. (b) 20. (b) 30. (d) 40. (c) 50. (b) 60. (c) 70. (a) 80. (b) 90. (a) 100. (a)

Q1.
Answer: c
Explanation:
● The National Statistical Office (NSO), a division of the Ministry of Statistics and Programme
Implementation (MoSPI) under the Government of India, is responsible for publishing the all India
Consumer Price Index (CPI).
● The NSO collects price data from selected markets and villages across India to calculate the CPI, which
measures the average change in prices of a fixed basket of goods and services consumed by urban and
rural households. The CPI is released monthly, with separate indexes for rural, urban and combined
populations.
Therefore, option (c) is the correct answer.
Relevance: The National Statistical Office (NSO) released the All India Consumer Price Index (CPI) and Consumer
Food Price Index (CFPI) for Rural (R), Urban (U) and Combined (C) for November 2023.

Q2.
Answer: b
Explanation:
● Virtual Digital Asset (VDA) refers to any information, code, number, or token (not being Indian currency
or foreign currency) generated through cryptographic means or otherwise. It can be transferred, stored
or traded electronically. The definition of VDA also specifically includes Non-Fungible Tokens (NFTs) or
any other token of a similar nature.
● VDA Service Providers in India are under anti-money laundering/counter-financing of terrorism
regulations. They are mandated to comply with the Prevention of Money Laundering Act (PMLA) 2002,
verify the identities of onboarded clients and maintain records of their financial positions and potentially
suspicious transactions. This obligation applies to all VDA Service Providers operating in India
irrespective of physical presence. So, statement 1 is not correct.

1
Vajiram & Ravi PowerUp Current Affairs Test Series - 2024
CA Test No. – 10 (CA3320)
.
● All VDA Service Providers operating in India and engaged in activities such as the exchange between
VDAs and fiat currencies were required to be registered with the Financial Intelligence Unit (FIU). So,
statement 2 is correct.
Therefore, option (b) is the correct answer.
Knowledge Box

● Financial Intelligence Unit – India (FIU-IND) was set by the Government of India as the
central national agency responsible for receiving, processing, analysing and
disseminating information relating to suspect financial transactions.
● It is also responsible for coordinating and strengthening efforts of national and
international intelligence, investigation and enforcement agencies in pursuing the
global efforts against money laundering and financing of terrorism.
● It is an independent body reporting directly to the Economic Intelligence Council (EIC)
headed by the Union Minister of Finance.
Relevance: Recently, the Financial Intelligence Unit India (FIU IND) issued show-cause notices to 9 offshore
Virtual Digital Asset Service Providers (VDA SPs) for “operating illegally” without complying with the provisions
of the Prevention of Money Laundering Act, 2002 (PMLA).

Q3.
Answer: d
Explanation:
● Kapdaganda Shawl refers to the traditional knitted shawls of the hill tribe Dongaria Kondh. Dongaria
Kondh is a Particularly Vulnerable Tribal Group (PVTG) residing in the Niyamgiri hills of Odisha. So, pair
1 is correctly matched.
● The Wancho Wooden Craft of Arunachal Pradesh is a rare ethnic art that has been practised by the
Wancho tribes for many generations. It is used both to decorate their drawing rooms and also as a gift
to beloved friends. So, pair 2 is not correctly matched.
● Kalonunia is one of the oldest indigenous aromatic rice varieties adopted in West Bengal. The paddy
field of Kalonunia appears entirely black, unlike the golden colour of normal paddy fields. So, pair 3 is
correctly matched.
● Kachchhi Kharek is the indigenous variety of dates of Kutch, Gujarat. Dates grown in Kutch are harvested
at the khalal stage, the stage when fruits have matured, accumulated sucrose and have turned red or
yellow but are still crisp. So, pair 4 is correctly matched.
Therefore, option (d) is the correct answer.
Relevance: Recently, the Geographical Indication (GI) Tags for multiple products from Odisha, Arunachal
Pradesh, West Bengal and Jammu and Kashmir were granted.

Q4.
Answer: d
Explanation:
PSLV Orbital Experimental Module (POEM):

2
Vajiram & Ravi PowerUp Current Affairs Test Series - 2024
CA Test No. – 10 (CA3320)
.
● It is a platform that will help perform in-orbit experiments using the final and otherwise discarded stage
of the Indian Space Research Organisation’s (ISRO’s) workhorse rocket, the Polar Satellite Launch Vehicle
(PSLV). The PSLV is a four-stage rocket where the first three spent stages fall back into the ocean and
the final stage (PS4), after launching the satellite into orbit, ends up as space junk. However, in PSLV-
C53 mission, the spent final stage will be utilised as a “stabilised platform” to perform experiments.
● POEM has a dedicated Navigation Guidance and Control (NGC) system for attitude stabilisation, which
stands for controlling the orientation of any aerospace vehicle within permitted limits. The NGC will act
as the platform’s brain to stabilise it with specified accuracy.
Therefore, option (d) is the correct answer.
Knowledge Box

● POEM will derive its power from solar panels mounted around the PS4 tank and a
Lithium-ion battery. It will navigate using four sun sensors, a magnetometer, gyros &
Navigation with Indian Constellation (NavIC).
● It carries dedicated control thrusters using Helium gas storage. It is enabled with a
telecommand feature.
● POEM platform has standard interfaces and packaging for power generation, telemetry,
tele-command stabilisation, orbital-station keeping and orbital manoeuvring. This
allows space agencies and private companies to design, develop and validate
experimental orbital payloads.
Relevance: Recently, two Indian Space startups successfully tested solutions onboard PSLV Orbital Experimental
Module (POEM).

Q5.
Answer: b
Explanation:
● Pangolins are also known as scaly anteaters. They are the only known mammals with large keratin
scales covering their skin. They are also toothless and nocturnal. When threatened, they curl up into a
tight ball. So, statement 1 is correct.
● Of the eight species of Pangolins found worldwide, two are found in India namely Indian Pangolin
(Manis crassicaudata) and Chinese Pangolin (Manis pentadactyla). The Indian Pangolin is found
throughout the country south of the Himalayas, excluding the North-Eastern region. It also occurs in
Bangladesh, Pakistan, Nepal and Sri Lanka. The Chinese Pangolin ranges through Assam and the eastern
Himalayas. So, statement 2 is not correct.
● The Indian Pangolin has a terminal scale on the ventral side of the tail which is absent in the Chinese
Pangolin. The colour of the scales of Indian Pangolin varies depending on the colour of the earth in its
surroundings. Its sticky tongue, which is longer than its body, is specially adapted for reaching and
lapping up insects in deep crevices. So, statement 3 is correct.
So, only two of the above statements are correct.
Therefore, option (b) is the correct answer.

3
Vajiram & Ravi PowerUp Current Affairs Test Series - 2024
CA Test No. – 10 (CA3320)
.
Knowledge Box

● Indian Pangolin is listed as ‘Endangered’ as per the International Union for


Conservation of Nature (IUCN) Red List of Threatened Species and Appendix I of the
Convention on International Trade in Endangered Species of Wild Fauna and Flora
(CITES).
Relevance: Recently, the Pangolin Conservation Project in Madhya Pradesh was deemed successful.

Q6.
Answer: c
Explanation:
● The agreement on the Prohibition of Attack against Nuclear Installations and Facilities was signed on
December 31, 1998, by the then Pakistani Prime Minister Benazir Bhutto and Indian Prime Minister Rajiv
Gandhi. The treaty came into force on January 27, 1991.
● The agreement mandates both countries to inform each other about any nuclear installations and
facilities to be covered under the agreement on the first of January of every calendar year, providing a
confidence-building security measure environment. So, statement 1 is correct.
● According to the agreement, the term ‘nuclear installation or facility’ includes nuclear power and
research reactors, fuel fabrication, uranium enrichment, iso-topes separation and reprocessing
facilities. It also includes any other installations with fresh or irradiated nuclear fuel and materials in
any form and establishments storing significant quantities of radioactive materials. So, statement 2 is
correct.
● The agreement refrains both countries from undertaking, encouraging or participating in, directly or
indirectly, any action aimed at causing the destruction of, or damage to, any nuclear installation or
facility in the other country.
Therefore, option (c) is the correct answer.
Relevance: India and Pakistan on January 1 exchanged a list of their nuclear installations under Agreement on
the Prohibition of Attack against Nuclear Installations and Facilities.

Q7.
Answer: b
Explanation:
● Chinnar Wildlife Sanctuary located in the rain shadow region of the Western Ghats is the second habitat
for the endangered Grizzled giant squirrel in India. The sanctuary has the unique thorny scrub forest
with xerophytic species.
● It is nestled in the hills of Kerala and is the only dedicated rehabilitation center for the Indian Star
Tortoise in the country. The south side of the sanctuary is covered by Eravikulam National park and the
north side is by Indira Gandhi wildlife sanctuary.
Therefore, option (b) is the correct answer.
Relevance: A team of researchers from Kerala has claimed to have spotted the presence of Eurasian Otter (Lutra
lutra) in the state for the first time at Chinnar Wildlife Sanctuary.

4
Vajiram & Ravi PowerUp Current Affairs Test Series - 2024
CA Test No. – 10 (CA3320)
.
Q8.
Answer: a
Explanation:
The Unlawful Activities (Prevention) Amendment Act (UAPA) was first enacted in 1967 and has been amended
several times since then. The Act was amended in 2019 to include provisions of designating an individual as
terrorist. Prior to this amendment, only organisations could be designated as terrorist organisations.
● The words “terror” or “terrorist” are not defined in the UAPA. However, it defines a “terrorist act” as
any act committed with intent to threaten or likely to threaten the unity, integrity, security, economic
security or sovereignty of India. It also includes any act committed with intent to strike terror or likely
to strike terror in the people or any section of the people in India or in any foreign country. So, statement
1 is not correct.
● The Central Government may designate an individual as a terrorist through a notification in the official
gazette and add his name to the Fourth Schedule to the UAPA. The Government is not required to give
an individual an opportunity to be heard before such a designation. So, statement 2 is not correct.
● Under the UAPA Amendment 2019, the Central Government set up a review committee comprising a
chairperson (a retired or sitting judge of a High Court) and three other members. The review committee
will be empowered to order the government to delete the name of the individual from the schedule
that lists “terrorists” if it considers the order to be flawed. So, statement 3 is correct.
So, only one of the above statements is correct.
Therefore, option (a) is the correct answer.
Relevance: Recently, a gangster was declared a designated terrorist by the Central Government under the
Unlawful Activities (Prevention) Act (UAPA), 2019.

Q9.
Answer: a
Explanation:
● The Reserve Bank of India recently released comprehensive guidelines on the measures to be put in
place by banks while classifying accounts and deposits as inoperative accounts and unclaimed deposits.
● A savings as well as current account should be treated as inoperative/dormant if there are no
transactions in the account for over a period of two years. The accounts which have not been operated
upon over a period of two years should be segregated and maintained in separate ledgers. Balances in
savings/current accounts which are not operated for 10 years, or term deposits not claimed within 10
years from date of maturity are classified as unclaimed deposits. So, statement 1 is not correct.
● For the purpose of classifying an account as ‘inoperative’, only customer induced transactions and not
bank induced transactions are considered. There may be instances where the customer has been given
a mandate like Standing Instructions (SI)/ auto-renewal instructions and there are no other operations
in the savings/current account or the term deposit. These transactions shall also be treated as customer
induced transactions. So, statement 2 is not correct.
● The new guidelines prohibit banks from classifying zero balance accounts as ‘inoperative’ even if they
remain non-operative for a period of more than two years. So, statement 3 is correct.
So, only one of the above statements is correct.
Therefore, option (a) is the correct answer.

5
Vajiram & Ravi PowerUp Current Affairs Test Series - 2024
CA Test No. – 10 (CA3320)
.
Q10.
Answer: b
Explanation:
Central Armed Police Forces (CAPFs):
● It is the collective name for central police organisations in India. These forces are responsible for internal
security and guarding the borders.
● The CAPFs refers to seven security forces which include the Assam Rifles, Border Security Force (BSF),
Central Industrial Security Force (CISF), Central Reserve Police Force (CRPF), Indo Tibetan Border Police
(ITBP), National Security Guard (NSG) and Sashastra Seema Bal (SSB). They all function under the
administrative control of the Union Ministry of Home Affairs. However, the operational control of the
Assam Rifles rests with the Union Ministry of Defence. So, statement 3 is correct.
● Amongst the CAPFs, the Assam Rifles, BSF, ITBP and SSB are Border Guarding Forces. The NSG is a
commando-trained force Organisation in India and is used for special operations. The CISF provides
security and protection to industrial undertakings and vital installations. The CRPF is deployed in aid of
civil power in matters relating to the maintenance of law and order, internal security and
counterinsurgency. The CAPFs are headed by officers from the Indian Police Services (IPS). So, statement
1 is not correct and statement 2 is correct.
So, only two of the above statements are correct.
Therefore, option (b) is the correct answer.
Knowledge Box

● Assam Rifles (AR): It is a central police and paramilitary organisation responsible for
border security, counter-insurgency and law and order in Northeast India. Its primary
role is to guard the Indo-Myanmar border. The AR is often nicknamed "Sentinels of the
North East" and "Friends of the Hill People". It is the oldest paramilitary force in India.
● Border Security Force: Its primary role is to guard the Indo-Pakistan and Indo-
Bangladesh borders. It is deployed both on the international border and the Line of
Control (LOC). It currently stands as the world's largest border-guarding force. It has
been termed as the First Line of Defence of Indian Territories.
● Central Industrial Security Force (CISF): It came into existence in 1969 to provide
integrated security cover to certain sensitive public sector undertakings. The CISF
security umbrella includes India’s most critical infrastructure facilities like nuclear
installations, space establishments, airports, seaports, power plants etc.
● Central Reserve Police Force (CRPF): It came into existence as the Crown
Representative's Police in 1939. It has played an increasingly large role in India's
general elections. The Central Reserve Police includes:
○ The Rapid Action Force (RAF), a 15-battalion anti-riot force trained to respond
to sectarian violence.
○ The Commando Battalion for Resolute Action (COBRA), a 10-battalion strong
anti-Naxalite force.

6
Vajiram & Ravi PowerUp Current Affairs Test Series - 2024
CA Test No. – 10 (CA3320)
.
● Indo-Tibetan Border Police (ITBP): It was established in 1962 in the aftermath of the
Indo-China war. It is deployed for guarding duties on the Indo-China border from
Karakoram Pass in Ladakh to Diphu Pass in Arunachal Pradesh.
● National Security Guard (NSG): Commonly known as Black Cats, it is a counter-
terrorism unit. It was founded under the National Security Guard Act, 1986 following
Operation Blue Star, to combat terrorist activities and protect states against internal
disturbances.
● Sashastra Seema Bal (SSB): It is a border guarding force of India deployed along its
borders with Nepal and Bhutan. The force was originally set up under the name Special
Service Bureau in 1963 in the aftermath of the Indo-China War to strengthen India's
border areas against enemy operations.
Relevance: Three battalions of the Border Security Forces (BSF) personnel are set to move across the border
from Odisha to Chhattisgarh to intensify anti-Maoist operations.

Q11.
Answer: a
Explanation:
Convention on the Prevention and Punishment of the Crime of Genocide:
● It was adopted in 1948 in response to the atrocities committed during World War II (1939 -45). It was
among the first United Nations conventions addressing humanitarian issues.
● The International Court of Justice (ICJ) recognises that the principles underlying the Convention are
principles which are recognised by civilised nations binding on States, even without any conventional
obligation.
● In the present Convention, genocide means any of the following acts committed with intent to destroy,
in whole or in part, a national, ethnical, racial or religious group, as such:
○ Killing members of the group
○ Causing serious bodily or mental harm to members of the group
○ Deliberately inflicting on the group conditions of life calculated to bring about its physical
destruction in whole or in part
○ Imposing measures intended to prevent births within the group. So, statement 1 is correct.
○ Forcibly transferring children of the group to another group
● The Convention states that genocide, whether committed in time of peace or in time of war, is a crime
under international law that the contracting parties undertake to prevent and punish.
● The Convention states that persons committing genocide shall be punished, whether they are
constitutionally responsible rulers, public officials or private individuals. So, statement 2 is not correct.
● As per the Convention, genocide shall not be considered as a political crime for the purpose of
extradition. The Contracting Parties pledge themselves in such cases to grant extradition in accordance
with their laws and treaties in force. So, statement 3 is not correct.
So, only one of the above statements is correct.
Therefore, option (a) is the correct answer.

7
Vajiram & Ravi PowerUp Current Affairs Test Series - 2024
CA Test No. – 10 (CA3320)
.
Relevance: Recently, South Africa has filed a genocide case against Israel at the International Court of Justice
(ICJ).

Q12.
Answer: c
Explanation:
Green Cover Index:
● It is a comprehensive pan-India estimation of green cover for the National Highways using high-
resolution satellite imagery. It will be developed by the National Remote Sensing Centre (NRSC) of the
Indian Space Research Organisation (ISRO).
● The index will witness the use of emerging technologies to enhance in-situ data collection, supplement
plantation management and monitoring, including performance audits conducted by the National
Highways Authority of India (NHAI).
● It will be a time-saving and cost-effective solution to generate a macro-level estimate along national
highways.
● The findings of the index will facilitate the comparison and ranking of various national highways for
timely and periodic intervention.
Therefore, option (c) is the correct answer.
Relevance: The National Highways Authority of India (NHAI) has recently signed a deal with the National Remote
Sensing Centre (NRSC) to develop and report a ‘Green Cover Index’.

Q13.
Answer: d
Explanation:
National Transit Pass System (NTPS):
● It is envisioned as a "One Nation-One Pass" regime, which will enable seamless transit across the
country. This initiative will streamline the issuance of timber transit permits by providing a unified,
online mode for tree growers and farmers involved in agroforestry across the country.
● It is designed for user convenience, featuring desktop and mobile applications for easy registration and
permit applications. Currently, transit permits are issued for transport of timber and forest produce
based on state-specific transit rules. Transit permits will be issued for tree species which are regulated,
while the users can self-generate No Objection Certificates for exempted species.
● Presently, 25 States and Union Territories have accepted the unified permit system, streamlining
interstate business operations for producers, farmers and transporters. This move is expected to provide
a significant impetus to the agroforestry sector.
Therefore, option (d) is the correct answer.
Relevance: Recently, the National Transit Pass System (NTPS) was launched to facilitate the seamless transit of
timber, bamboo and other forest produce across the country.

Q14.
Answer: c
Explanation:

8
Vajiram & Ravi PowerUp Current Affairs Test Series - 2024
CA Test No. – 10 (CA3320)
.
● Recombinant DNA (rDNA) technology involves procedures for analyzing or combining DNA fragments
from one or several organisms including the
introduction of the rDNA molecule into a cell for
its replication, or integration into the genome
of the target cell.
● Traditional hybridisation procedures used in
plant and animal breeding, very often lead to
inclusion and multiplication of undesirable
genes along with the desired genes. The
techniques of genetic engineering which
include the creation of rDNA, the use of gene
cloning and gene transfer, overcome this
limitation and allow the isolation and
introduction of only one or a set of desirable
genes without introducing undesirable genes
into the target organism.
● It can be accomplished only with the presence of key tools, i.e., restriction enzymes, polymerase
enzymes, ligases, vectors and the host organism.
○ Restriction Enzymes recognise specific DNA sequences and cleave the DNA at the precise
location. Examples include CRISPR Cas9 and Zinc-Finger Nuclease (ZFN). So, point 1 is correct.
○ Selectable markers are used to distinguish transformed cells from non-transformed ones.
Examples include Antibiotic resistance genes, herbicide-resistance genes, etc. So, point 2 is
correct.
○ Vectors are used as carriers to introduce foreign DNA into a host cell. Examples include Plasmids,
Yeast Artificial Chromosomes, Bacterial Artificial Chromosomes etc. So, point 4 is correct.
● Cytokine barriers are virus-infected cells that secrete proteins called interferons which protect non-
infected cells from further viral infection. They are not among the tools of rDNA technology. So, point 3
is not correct.
Therefore, option (c) is the correct answer.
Knowledge Box

● Applications of rDNA technology include Recombinant human insulin, human growth


hormone, hepatitis B vaccine, Golden rice and Herbicide and insect-resistant crops. It
also serves as a tool for identification, mapping and sequencing of genes.
Relevance: As per a recent market forecast, the recombinant DNA technology market is set to grow in the
coming year.

Q15.
Answer: c
Explanation:

9
Vajiram & Ravi PowerUp Current Affairs Test Series - 2024
CA Test No. – 10 (CA3320)
.
● A rock glacier is a mass of rock, ice, snow, mud and water that moves slowly down a mountain under
the influence of gravity. Unlike an ice glacier, rock glaciers usually have very little ice visible at the surface.
The rock glacier might consist of a mass of ice covered by rock debris, or it might consist of a mass of
rock with interstitial ice. So, statement 1 is correct.
● Rock glaciers typically form in mountainous regions where there is a combination of permafrost, rock
debris, and ice. One common scenario involves a pre-existing glacier that accumulates debris and rocks
as it moves. Over time, if the glacier recedes or thaws, the debris-covered ice can transform into a rock
glacier. These are classified as ‘active’ or ‘relict’ to indicate the status of permafrost within them,
identified by the appearance of the rock surfaces. So, statement 2 is correct.
Therefore, option (c) is the correct answer.
Relevance: A recent study mapped that over 100 active permafrost structures identified in Jhelum basin, can
cause catastrophic disasters in future.

Q16.
Answer: c
Explanation:
● The Acceptance of Necessity (AoN) Accord is a crucial stage in the Indian defence procurement process.
It signifies the government's approval to move forward with the procurement of a specific military
equipment or system. The AoN Accord is the first step of the acquisition process and establishes that
the military has an urgent need for the equipment that is being cleared. It is a formal document issued
by the Defence Acquisition Council (DAC), chaired by the Union Minister of Defence.
o The DAC is the highest decision-making body of the Ministry of Defence on procurement. It
ensures expeditious procurement of the approved requirements of the armed forces. The Union
Defence Minister is the chairman of DAC. Its members include the Chief of Defence Staff (CDS)
and chiefs of the Army, Navy and Air Force.
Therefore, option (c) is the correct answer.
Relevance: The Ministry of Defence (MoD) had cleared the purchase of DRDO-developed weapons systems
worth more than Rs 1.42 trillion.

Q17.
Answer: b
Explanation:
● The temple was built in 1792 with the efforts of Sardar Rangarao Odhekar. It is said that Sardar Odhekar
dreamt of a black-coloured statue of Lord Ram in the Godavari and recovered the statues from the river
and built the temple. The place where statues were found was named Ramkund. So, statement 1 is not
correct.
● The Kalaram temple derives its name from a black statue of the Lord Ram — Kala Ram translates
literally to “Black Ram”. The sanctum sanctorum has statues of Ram, Sita and Lakshman and a black idol
of Hanuman at the main entrance. So, statement 2 is correct.
● The Kalaram Temple Entry Satyagraha was initiated under the guidance of Dr B.R. Ambedkar in 1930.
From 3rd March, 1930, some 15,000 Dalit followers of Ambedkar initiated a picketing around the
temple. The satyagraha went on for another three years, but the temple doors remained closed to the

10
Vajiram & Ravi PowerUp Current Affairs Test Series - 2024
CA Test No. – 10 (CA3320)
.
Dalits. Even though the Kalaram Temple Entry Satyagraha was not successful, Dr. Ambedkar's deeper
objectives were achieved as oppressed caste members from every strata participated and contributed
to the satyagraha. So, statement 3 is correct.
So, only two of the above statements are correct.
Therefore, option (b) is the correct answer.
Relevance: Recently, the Prime Minister visited the Kalaram Mandir situated on the banks of the Godavari in
Maharashtra.

Q18.
Answer: c
Explanation:
● Recently scientists have examined the existence of high-frequency plasma waves in the Martian plasma
environment by making use of the high-resolution electric field data from the Mars Atmosphere and
Volatile Evolution Mission (MAVEN) spacecraft of NASA.
● Plasma waves are identified as the short-time scale fluctuations in the electric and magnetic field
observations. These plasma waves play an important role in the energization and transport of the
charged particles in the Earth’s magnetosphere, a magnetic field cavity around the Earth. So,
statements 1 and 4 are correct.
○ Mars does not have any intrinsic magnetic field therefore the high-speed solar wind coming from
the Sun interacts directly with the Mars atmosphere, like an obstacle in the flow.
● Some of the plasma waves like electromagnetic ion cyclotron waves act as a cleaning agent for the
Earth’s radiation belt, which is hazardous to our satellites. Knowing this scenario, researchers are
curious to understand the existence of various plasma waves in the vicinity of unmagnetized planets like
Mars. So, statement 2 is correct.
● Plasma waves can occur in both low and high frequencies and their characteristics depend on various
factors, including the properties of the plasma medium in which they propagate. So, statement 3 is not
correct.

So, only three of the above statements are correct.


Therefore, option (c) is the correct answer.
11
Vajiram & Ravi PowerUp Current Affairs Test Series - 2024
CA Test No. – 10 (CA3320)
.
Knowledge Box

Mars Atmosphere and Volatile Evolution Mission (MAVEN):


● It was launched by the National Aeronautics and Space Administration (NASA) in 2013
to study the Mars atmosphere.
● Its goal was to explore the planet’s upper atmosphere, ionosphere, and interactions
with the Sun and solar wind.
● Scientists use MAVEN data to explore the loss of volatile compounds—such as Carbon
dioxide (CO2), Nitrogen (N2) and Water (H2O)—from the Martian atmosphere to space.
○ Understanding atmospheric loss gives scientists insight into the history of Mars’
atmosphere and climate, liquid water, and planetary habitability.
Relevance: High-frequency waves detected in the Martian Upper Atmosphere could help understand plasma
processes over Mars.

Q19.
Answer: d
Explanation:
● The Supreme Court Legal Services Committee (SCLSC) was constituted under the Legal Services
Authorities Act, 1987, to provide “free and competent legal services to the weaker sections of society”,
in cases falling under the top court’s jurisdiction. So, statement 1 is not correct.
● The Act states that the Central Authority (the National Legal Services Authority or NALSA) shall
constitute the committee. It consists of a sitting SC judge, who is the chairman, along with other
members possessing the experience and qualifications prescribed by the Centre. Both the chairman and
other members will be nominated by the Chief Justice of India (CJI). Further, the CJI can appoint the
Secretary to the Committee. So, statements 2 and 3 are not correct.
● Besides this, the NALSA Rules, 1995, entails the numbers, experience and qualifications of the SCLSC
members. Under the 1987 Act, the Centre is empowered to make rules in consultation with the CJI, by
notification, to carry out the provisions of the Act
Therefore, option (d) is the correct answer.
Relevance: Justice Gavai was recently nominated as the Chairman of the Supreme Court Legal Services
Committee.

Q20.
Answer: b
Explanation:
● Researchers have recently identified a new class of antibiotics, ‘Zosurabalpin’, with the potential to
tackle a drug-resistant bacterium, Acinetobacter baumannii. Zosurabalpin was found to be effective
against CRAB (carbapenem-resistant Acinetobacter baumannii) induced pneumonia and sepsis in
mouse models.
● It inhibits a key process, transport of the molecule lipopolysaccharide (LPS), by inhibiting a complex of
proteins. This complex was essential for transporting LPS to the bacterial surface to create the outer-

12
Vajiram & Ravi PowerUp Current Affairs Test Series - 2024
CA Test No. – 10 (CA3320)
.
membrane structure of Gram-negative bacteria. Zosurabalpin blocks LPS transport and the abnormal
build-up of LPS in the cell kills the bacterium.
Therefore, option (b) is the correct answer.
Relevance: Recently, researchers have identified a new class of antibiotics, Zosurabalpin, with the potential to
tackle a drug-resistant bacterium, Acinetobacter baumannii.

Q21.
Answer: c
Explanation:
● Sisal is a xerophytic, monocarp, semi-perennial leaf fiber
producing plant. It can survive in dry conditions, produces only
one flower in its lifetime and has leaves that can be used to
make fibres. So, statement 1 is correct.
● Sisal leaves are not short and slender; they are thick and
fleshy. Leaves are often covered with a waxy layer. Its leaves
grow up to 2 m long. The lifespan of a sisal plant is about 7-10
years, during which it produces 200-250 usable leaves.
● Sisal grows well in arid and semi-arid regions with a warm
climate. It is well-suited for tropical and subtropical climates.
Sisal prefers temperatures between 25 to 30 degrees Celsius. Sisal is drought-resistant but can benefit
from periodic rainfall. It is well-adapted to regions with low and erratic rainfall. So, statement 2 is
correct.
● Sisal is a versatile plant that can be used to make various products, such as paper, cloth, footwear, hats,
bags, carpets, geotextiles and dartboards. Each leaf has about a thousand fibres that can be used to
make ropes, paper and cloth. Now, it could be used to make a highly absorbent material as well. So,
statement 3 is correct.
○ The ancient Aztec and Mayan civilizations were perhaps the first to begin making paper out of
sisal leaves.
So, all three of the above statements are correct.
Therefore, option (c) is the correct answer.
Relevance: A team at Stanford University has found the absorption capacity of a sisal-based material to be
higher than those in commercial menstrual pads.

Q22.
Answer: c
Explanation:
● The thylakoid membrane is a component of the chloroplast in plant cells and is directly involved in the
process of photosynthesis. It is the site where light-dependent reactions occur, capturing sunlight and
converting it into chemical energy, which is used to produce Adenosine Triphosphate (ATP) and
Nicotinamide Adenine Dinucleotide Phosphate (NADPH) for further stages of photosynthesis.
● Thylakoid was originally present in ancient cyanobacteria. It contributes to oxygen release and is crucial
for life.

13
Vajiram & Ravi PowerUp Current Affairs Test Series - 2024
CA Test No. – 10 (CA3320)
.
Therefore, option (c) is the correct answer.
Relevance: Recently, the discovery of 1.75-billion-year-old fossil thylakoids in Australia suggests their existence
far earlier than thought.

Q23.
Answer: c
Explanation:
● The PRITHvi VIgyan (PRITHVI) is an initiative of the Union Ministry of Earth Sciences. It encompasses
five ongoing sub-schemes namely:
○ Atmosphere & Climate Research-Modelling Observing Systems & Services (ACROSS). So, point 1
is correct.
○ Ocean Services, Modelling Application, Resources and Technology (O-SMART). So, point 2 is
correct.
○ Polar Science and Cryosphere Research (PACER). So, point 3 is correct.
○ Seismology and Geosciences (SAGE). So, point 4 is correct.
○ Research, Education, Training and Outreach (REACHOUT)
● The major objectives of the Scheme are:
○ Augmentation and sustenance of long-term observations of the atmosphere, ocean, geosphere,
cryosphere and solid earth to record the vital signs of the Earth System and change
○ Development of modelling systems for understanding and predicting weather, ocean and
climate hazards and understanding the science of climate change ·
○ Exploration of polar and high seas regions of the Earth towards the discovery of new phenomena
and resources
○ Development of technology for exploration and sustainable harnessing of oceanic resources for
societal applications
○ Translation of knowledge and insights from Earth systems science into services for societal,
environmental and economic benefit.
● Synergistic Training program Utilising the Scientific and Technological Infrastructure (STUTI) program
has been designed to cater to human resources and its capacity building through open access to Science
and Technology infrastructure across the country. It is a scheme of the Department of Science and
Technology. It is not a sub-scheme of PRITHVI scheme. So, point 5 is not correct.
Therefore, option (c) is the correct answer.
Relevance: Recently, the Union Cabinet has sanctioned the Prithvi Vigyan (Prithvi) scheme.

Q24.
Answer: c
Explanation:
● The drainage basin of the Congo River spans multiple countries, including the Democratic Republic of
Congo, Republic of Congo, Central African Republic, Angola, Cameroon and Tanzania.

14
Vajiram & Ravi PowerUp Current Affairs Test Series - 2024
CA Test No. – 10 (CA3320)
.
● The River, after its extensive journey through Central Africa, drains into the Atlantic Ocean. It forms a
vast estuary as it meets the Atlantic Ocean, contributing to the ocean's waters along the west coast of
Africa.
● Congo River is the world's deepest recorded river at 720
feet (220 meters) deep in parts — too deep for light to
penetrate. It is the second-longest river in Africa (Africa's
Nile River is the longest river in the world).
Therefore, option (c) is the correct answer.
Relevance: Recently, Congo River has risen to its highest level in
more than 60 years, causing flooding throughout the Democratic
Republic of Congo.

Q25.
Answer: a
Explanation:
● The Gulf of Aden is a deepwater basin that forms a natural sea link between the Red Sea and the
Arabian Sea. It is named after the seaport of Aden, in southern Yemen. The Gulf is situated between the
coasts of Arabia and the Horn of Africa. It narrows towards the west side into the Gulf of Tadjoura. Its
eastern geographic limits are defined by the meridian of Cape Guardafui. So, statement 1 is correct.
● It is bounded to the south by the Somali and the Socotra Islands, north by Yemen, east by the Arabian
Sea and west by Djibouti. The gulf is connected to the
Somali Sea to the south by the Guardafui Channel.
The Strait of Bab-el-Mandeb connects it to the Red
Sea. So, statement 2 is correct.
● The Gulf of Aden is mainly shared by three countries
namely Yemen, Somalia and Djibouti. Their exclusive
economic zones extend into the water body. The
main commodity transported through the Gulf is the
Persian Gulf oil. So, statement 3 is not correct.
● The dominant relief feature of the gulf’s terrain is the
Sheba Ridge, an extension of the Indian Ocean ridge
system, which extends along the middle of the gulf.
The rough topography of the ridge includes a well-
defined median valley that is continually offset by
faults running approximately northeast to southwest.
The largest of these faults forms the Alula-Fartak Trench, which is found in the gulf’s maximum recorded
depth.
Therefore, option (a) is the correct answer.
Relevance: Recently, Houthi militants had hit a vessel of the United States with a missile in the Gulf of Aden.

15
Vajiram & Ravi PowerUp Current Affairs Test Series - 2024
CA Test No. – 10 (CA3320)
.
Q26.
Answer: a
Explanation:
● The Bugun Liocichla bird is primarily found in the Eaglenest Wildlife Sanctuary in Arunachal Pradesh.
It was discovered there by an astrophysicist, Ramana Athreya, in 2006.
● It appears to favour dense forest edges and degraded forest at 2060-2340 metres. It clambers about in
lower levels of forest, typically in pairs or small groups, occasionally with otherwise uniform flocks of
other species.
● It is closely related to the Emei Shan liocichla, Liocichla omeiensis, a species of Liocichla endemic to
China.
● It is listed as ‘Critically Endangered’ in the International Union for Conservation of Nature (IUCN) Red
List of Threatened Species.
Therefore, option (a) is the correct answer.
Knowledge Box

Eaglenest Wildlife Sanctuary:


● It is one of the last strong-holds for species such as the Asian yew (Taxus wallichiana)
and the Anise tree (Illicium griffithi).
● The critically endangered Gymnocladus assamicus (Himalayan soap pod tree) which
is endemic to north-eastern India, is also found here.
● The recently discovered Arunachal macaque is also found in Eaglenest, as are the
rare spotted linsang, red panda, slow loris, clouded leopard and golden cat.
Relevance: A study in Arunachal Pradesh reveals how logging and climate change impact montane birds
including Bugun Liocichla.

Q27.
Answer: c
Explanation:
● The annual Periodic Labour Force Survey (PLFS) provides a timely
assessment of the nation’s economy and aids in making better-
informed policy decisions.
● The 2022-23 PLFS has highlighted the decent growth reported in
India’s total employment, a rising female Labour Force Participation
Ratio (LFPR) and a higher share of self-employment, often with some
conjecture on informal-sector performance.
● While the Labour Force Participation Ratio (LFPR) and Worker
Population Ratio (WPR) of males aged above 15 years have increased
in 2022-23, they have fallen for all ages and dropped sharply for the
prime working age group of 30-59 years. Details suggest that while the
LFPR and WPR for females have improved across urban and rural

16
Vajiram & Ravi PowerUp Current Affairs Test Series - 2024
CA Test No. – 10 (CA3320)
.
areas, these ratios for males have declined in both urban and rural India, with a substantial fall in the
latter. So, statement 1 is correct.
● The PLFS 2022-23 shows an unemployment rate of 10% in the age group of 15-29 years, as compared
to an unemployment rate of 3.2% across all age groups. This, of course, is an important difference, but
it should be noted that the unemployment rate in the age group of 15-29 has seen a very sharp decline
from 17.8% in 2017-18. So, statement 2 is not correct.
● The majority (up to 85%) of the increase in female employment is in the agriculture sector and self-
employment category. So, statement 3 is correct.
Therefore, option (c) is the correct answer.

Q28.
Answer: d
Explanation:
● Proba-3 is a dual probe technological demonstration mission by the European Space Agency (ESA)
devoted to high precision formation flying to achieve scientific coronagraphy. It is part of the series of
PROBA satellites that are being used to validate new spacecraft technologies and concepts while also
carrying scientific instruments.
● The mission consists of two independent, three-axis stabilised spacecraft: the Coronagraph Spacecraft
(CSC) and the Occulter Spacecraft (OSC). The OSC will block the Sun’s direct light for the CSC, allowing it
to image the faint solar corona in visible, ultraviolet and polarised light for many hours at a time.
● The mission is expected to launch in September 2024 on a PSLV-XL rocket from India. The mission
duration is 2 years (nominal). The main scientific objective is to observe the corona down to about 1.1
solar radius in the visible wavelength range.
Therefore, option (d) is the correct answer.
Relevance: Proba-3 is expected to launch in September 2024 on a PSLV-XL rocket from India.

Q29.
Answer: c
Explanation:
● The Telecommunications Act, 2023 creates a Digital Bharat Nidhi, a non-lapsable fund, for the
development of telecommunication infrastructure and services. The Universal Service Obligation Fund
(USOF) is retained as Digital Bharat Nidhi, allowing its use for research and development. So, statement
1 is correct.
● The Act requires authorisation from the Central Government (not the State Governments) for providing
telecommunication services, establishing or operating telecommunication networks and possessing
radio equipment. So, statement 2 is not correct.
● It empowers the Central Government to reclaim unutilized spectrum and provide options for sharing,
trading, leasing and voluntary surrender of unused spectrum. It also specifies the fees and charges for
spectrum usage. So, statement 3 is correct.
● Over-the-top (OTT) services are excluded from the Telecommunications Act, 2023 and their regulation
will fall under the potential Digital India Bill, 2023. So, statement 4 is correct.
So, only three of the above statements are correct.
17
Vajiram & Ravi PowerUp Current Affairs Test Series - 2024
CA Test No. – 10 (CA3320)
.
Therefore, option (c) is the correct answer.
Relevance: The Parliament passed the Telecommunications Act, 2023 to replace the Indian Telegraph Act, 1885,
the Indian Wireless Telegraphy Act, 1933 and the Telegraph Wires (Unlawful Possession) Act, 1950.

Q30.
Answer: d
Explanation:
● Mulya Pravah 2.0 are guidelines issued by the University Grants Commission (UGC) in India, aiming to
instil human values and professional ethics in higher education institutions. It is a modified version of
the 2019 Mulya Pravah guideline.
● Some of the main objectives of Mulya Pravah 2.0 are:
○ To inculcate deep respect for fundamental duties and constitutional values among the students,
teachers and staff of higher education institutions.
○ To develop a culture of integrity, accountability, transparency and inclusiveness in higher
education institutions and to prevent corruption, discrimination and malpractices.
○ To foster a sense of belongingness, harmony, sustainability and global citizenship among the
stakeholders of higher education institutions.
○ To reinforce human values and professional ethics through curriculum, pedagogy, assessment,
research, extension and governance in higher education institutions.
Therefore, option (d) is the correct answer.
Relevance: The University Grants Commission (UGC) is pushing for values and ethics in higher education through
guidelines like Mulya Pravah 2.0.

Q31.
Answer: c
Explanation:
India-United Arab Emirates (UAE) relations:
● Economic and Commercial cooperation between India and the UAE is a key aspect of the overall bilateral
relationship. As per the Economic Survey 2022-23, the United Arab Emirates was India’s fourth-largest
investor and third-largest trading partner. The UAE was also the second largest export destination for
India after the United States of America (USA). So, statement 1 is correct and statement 2 is not correct.
● India and the UAE have signed the Comprehensive Economic Partnership Agreement (CEPA) in 2022. It
has also significantly decreased tariffs, streamlined customs procedures and introduced a series of
robust and transparent regulations that support rules-based competition. So, statement 3 is correct.
● The UAE has become a member of the Brazil, Russia, India, China and South Africa (BRICS) grouping in
2024. So, statement 4 is correct.
Therefore, option (c) is the correct answer.
Knowledge Box

● The BRICS bloc of top emerging economies had recently announced the induction of
five full members as part of an attempt to expand its strategic heft against the backdrop
of Western dominance in world affairs.

18
Vajiram & Ravi PowerUp Current Affairs Test Series - 2024
CA Test No. – 10 (CA3320)
.
● It has become a 10-nation body with Egypt, Ethiopia, Iran, Saudi Arabia and the United
Arab Emirates (UAE) joining it as new members.
● In 2023, the top BRICS leaders approved a proposal to admit six countries, including
Argentina, into the bloc with effect from January 1. However, Argentina's new
President announced the withdrawal from becoming a member of the BRICS.
Relevance: Recently, the United Arab Emirates (UAE) has become a member of the BRICS grouping.

Q32.
Answer: b
Explanation:
● A Protection & Indemnity (P&I) Club is a type of mutual insurance association that provides liability
insurance for shipowners, operators and charterers. They are an essential component of the maritime
insurance industry, offering coverage beyond the scope of traditional hull and machinery insurance.
● A P&I Club is a non-profit, non-governmental association that provides marine insurance to its
members. Members include ship owners, operators, charterers and seafarers. P&I clubs provide risk
pooling, information and representation for their members. Unlike marine insurance companies, which
report to their shareholders, P&I Clubs only report to their members.
Therefore, option (b) is the correct answer.
Knowledge Box

● India plans to set up its own Protection and Indemnity (P&l) entity—the India Club—
for providing insurance to ships operating in Indian coasts and waterways.
○ The initiative is expected to benefit domestic fleet owners and reduce
dependence on foreign entities for insurance coverage.
○ The India Club will operate as a cooperative, pooling fees from members to
provide insurance cover for unforeseen situations such as cargo damage, war
damages and environmental risks
Relevance: India is planning to set up its own Protection and Indemnity (P&l) entity—the India Club.

Q33.
Answer: b
Explanation:
● Rudram Missile is an air-to-surface missile, designed and developed by the Defence Research and
Development Organisation (DRDO). It is the first indigenous anti-radiation missile of the country. Once
the missile locks on the target, it is capable of striking accurately even if the radiation source switches
off in between. Integrated with SU-30 MkI aircraft, it has a capability of varying ranges based on the
launch conditions. So, pair 1 is not correctly matched.
● The AKASH-NG (New Generation) is a medium-range, mobile surface-to-air missile defence system. It
can intercept high-speed, agile aerial threats with a range of around 80 km. It also includes an
indigenously developed radio frequency seeker, launcher and multi-function radar. So, pair 2 is correctly
matched.
19
Vajiram & Ravi PowerUp Current Affairs Test Series - 2024
CA Test No. – 10 (CA3320)
.
○ The Akash Weapon System has been indigenously designed and developed by the DRDO and
produced by defence public sector units.
● Nirbhay Missile, developed by DRDO, is a long-range, all-weather, subsonic cruise missile that can carry
conventional or nuclear warheads. It has a range of up to 1,500 km and a speed of 0.9 Mach.It has a
ring laser gyro inertial navigation system and a radio altimeter for guidance. So, pair 3 is correctly
matched.
● Pralay Missile is a quasi-ballistic surface-to-surface missile developed by DRDO. It has a range of 150 to
500 km and a speed of Mach 5. It is propelled by a solid-propellant rocket motor and has a warhead
weight of 500 kg. It can be launched from a mobile launcher or a transport vehicle. So, pair 4 is not
correctly matched.
So, only two of the above pairs are correctly matched.
Therefore, option (b) is the correct answer.
Knowledge Box

● A quasi-ballistic missile (also called a semi-ballistic missile) is a category of missile


that has a low trajectory and/or is largely ballistic but can perform manoeuvres in
flight or make unexpected changes in direction and range.
● Pralay is India's first tactical quasi-ballistic missile.
Relevance: The Defence Research and Development Organisation (DRDO) recently conducted a successful test
of the New Generation AKASH missile off the Odisha coast.

Q34.
Answer: c
Explanation:
● Magnetic cooling effect (MCE) is defined as the reversible temperature change of a magnetic material
when it is subjected to an externally applied magnetic field. In the magnetic refrigeration cycle, a
magnetic field is applied to the magnetic material under an adiabatic process (no exchange of heat with
the surroundings). Initially, randomly oriented magnetic moments get aligned along the external
magnetic field, resulting in the heating of the magnetic material. This heat is transferred from the
material to the ambience.
● When the magnetic field is removed during adiabatic demagnetization, the magnetic moments of the
material become randomised, resulting in a decrease in temperature below the ambient temperature.
This process causes the material to absorb heat from the surrounding heat-transfer medium. So,
statement 1 is correct.
● It offers an energy-efficient and environment-friendly cooling technology as an alternative to the
vapour-cycle refrigeration technology in use today. Hence efforts are on to fabricate magnetic
refrigerators for household, industrial, and technological applications. So, statement 2 is correct.
Therefore, option (c) is the correct answer.
Relevance: Researchers at SN Bose National Centre have found a new alloy that can act as an effective magnetic
refrigerant that can be an alternative cooling agent for minimising greenhouse gas emissions.

Q35.
20
Vajiram & Ravi PowerUp Current Affairs Test Series - 2024
CA Test No. – 10 (CA3320)
.
Answer: b
Explanation:
● NUTEC Plastics is the flagship initiative of the International Atomic Energy Initiative (IAEI) to address
the global challenge of plastic pollution. It brings together countries and partners from around the world
to fight plastic pollution on two fronts: at point of source, by introducing new technologies to improve
plastic recycling; and in the ocean, where the bulk of plastic waste ends up.
● Through the IAEA’s NUTEC Plastics initiative, 63 countries are participating in marine monitoring of
microplastics, and 30 around the world are developing innovative recycling technology.
● Nuclear science and technology can play a key role on both fronts. Irradiation can be used to treat
existing plastics and to render them fit for reuse – extending current recycling potential and enabling a
wider and higher-value reuse – and nuclear science is used to identify, trace and monitor plastics in the
ocean, particularly microplastics.
Therefore, option (b) is the correct answer.
Relevance: A new joint investigation was recently launched by the International Atomic Energy Agency and the
Argentine Antarctic Institute (IAA) as part of the IAEA's NUTEC Plastics Initiative.

Q36.
Answer: a
Explanation:
● The Ministry of Defence has finalised a new rotational plan for Republic Day parade tableaux selection.
As per a new agreement, a new arrangement has been made where every State and Union Territory
(UT) will get an opportunity to present their tableaux at the Republic Day parade within a three-year
cycle (2024-2026). So, statement 2 is not correct.
● A theme is decided every Republic Day and the States have to present a proposal with design and format
on the particular theme. Not just States, Ministries and various Government departments including the
military, National Service Scheme and others also present tableau. The theme for the tableaux at the
Republic Day Parade 2024 was 'Bharat - The Mother of Democracy'. So, statement 1 is not correct.
● The selection of tableaux for Republic Day is taken care of by the Union Ministry Of Defence. They are
also evaluated and awarded prizes based on their presentation and architecture. So, statement 3 is
correct.
○ A committee, formed by the Ministry, evaluates the proposal. Initially, the proposals and the
design sketch/blueprint are assessed.
○ Selection depends upon a combination of factors including but not limited to visual appeal,
impact on the masses, idea/theme of the tableaux, degree of detailing involved in the tableaux,
music accompanying the tableaux, local artists used etc.
So, only one of the above statements is correct.
Therefore, option (a) is the correct answer.
Relevance: The Union Ministry of Defence has recently presented a new rollover plan for Republic Day parade
tableaux selection.

Q37.
Answer: a

21
Vajiram & Ravi PowerUp Current Affairs Test Series - 2024
CA Test No. – 10 (CA3320)
.
Explanation:
Sidersaura Marae:
● It is a new species of sauropod dinosaur that has been discovered in Argentina. It was a long-necked,
duck-beaked creature that went extinct 90 million years ago during the Cenomanian age of the Late
Cretaceous epoch. It has been named because of its star-shaped tail bones.
● These dinosaurs are distinguished from other sauropods by their distinctive teeth. It had a very long
tail. Its skull bones are robust, unlike those of the rest of its closest relatives. It has a hole in the roof of
the skull.
Therefore, option (a) is the correct answer.
Relevance: Palaeontologists in Argentina recently found fossil fragments of a new species of dinosaur named
Sidersaura marae.

Q38.
Answer: b
Explanation:
Einstein Probe (EP):
● It is a satellite which uses new X-ray detection technology inspired by the functioning of a lobster eye.
It was launched by China by a Long March-2C carrier rocket from the Xichang Satellite Launch Centre.
So, statement 1 is not correct.
○ Since black holes and gravitational waves are predictions made by Einstein's theory of general
relativity, the satellite is named after him.
○ It is shaped like a lotus into space to observe mysterious transient phenomena in the universe.
The 12 'petals' are 12 modules consisting of Wide-field X-ray Telescopes (WXT), while the two
'stamens' consist of two modules of Follow-up X-ray Telescopes (FXT).
● These telescopes form a space observatory which is aimed at helping scientists capture the first light
stemming from supernova explosions, search for and pinpoint X-ray signals accompanying gravitational
wave events. They aim to discover dormant black holes and other faint transient and variable celestial
objects at the far reaches of the universe. Neutron stars, white dwarfs and gamma-ray bursts are also
observation targets of this mission. So, statement 2 is correct.
Therefore, option (b) is the correct answer.
Relevance: Recently, the Einstein Probe was launched by China.

Q39.
Answer: c
Explanation:
Meissner effect:
● It refers to the expulsion of a magnetic field from the interior of a material that is in the process of
becoming a superconductor. It loses its resistance to the flow of electrical currents when cooled below
a certain temperature. This temperature is called the transition temperature which is usually close to
absolute zero. The Meissner effect is a property of all superconductors.
● Generally, ranges of intermediate magnetic-field strengths, which are present during cooling, produce a
partial Meissner effect.

22
Vajiram & Ravi PowerUp Current Affairs Test Series - 2024
CA Test No. – 10 (CA3320)
.
○ Some superconductors, called type I (tin and mercury) can be made to exhibit a complete
Meissner effect by eliminating various chemical impurities and physical imperfections.
○ Type II superconductors (vanadium and niobium) exhibit only a partial Meissner effect at
intermediate magnetic-field strengths no matter what their geometrical shape or size. Type II
superconductors show decreasing expulsion of the magnetic field as its strength increases.
Therefore, option (c) is the correct answer.
Knowledge Box

● LK-99 is a compound of copper, lead, phosphorus and oxygen. It is speculated to be


the first superconductor that works at room temperature and ambient pressure.
● The Eliza effect refers to people’s tendency to falsely attribute human thought
processes and emotions to an Artificial Intelligence system, thus believing that the
system is more intelligent than it is. The phenomenon was named after ELIZA, a chatbot.
● The Disposition effect refers to our tendency to prematurely sell assets that have
made financial gains while holding on to assets that are losing money.
● The term Lotus effect refers to water repellency and self-cleaning properties. The
origin of this expression has to do with the lotus flower, which is a scientific mystery
because it has not been possible to discover why it does not get wet.
Relevance: Recently, researchers in Asia reported checking for signs of superconductivity in newly synthesised
samples of LK-99.

Q40.
Answer: c
Explanation:
● Dark matter is a mysterious form of matter that makes up most of the mass of the universe, but does
not interact with light or other electromagnetic radiation. Scientists infer its existence from its
gravitational effects on visible matter, such as stars and galaxies. In the current universe, nearly 70 per
cent constitutes dark energy whereas 25 per cent is dark matter – about both of which there is scanty
knowledge, till date.
● Cold Dark Matter (CDM) is a hypothetical form of dark matter that is characterised by its weak
interactions with ordinary matter. It does not interact strongly via electromagnetic or strong nuclear
forces, which makes it challenging to detect directly. So, statement 1 is correct.
● CDM moves slowly compared to the speed of light, giving it a vanishing equation of state. The term
“cold” refers to the low kinetic energy of the particles. So, statement 2 is correct.
● CDM is thought to have been present in the universe since the very beginning and has influenced the
formation and evolution of cosmic structures, such as galaxies and clusters. It allows structure to grow
hierarchically, with small objects collapsing first and merging to form larger ones. So, statement 3 is
correct.
So, all three of the above statements are correct.
Therefore, option (c) is the correct answer.

23
Vajiram & Ravi PowerUp Current Affairs Test Series - 2024
CA Test No. – 10 (CA3320)
.
Knowledge Box

Other Types of Dark Matter:


● Warm Dark Matter (WDM) is a hypothetical type of dark matter that moves faster
than CDM, but slower than the speed of light, giving it a non-negligible equation of
state.
● Hot Dark Matter (HDM) is a hypothetical type of dark matter that moves close to
the speed of light, giving it a high equation of state.
Relevance: Scientists have found a new approach to explore Cold Dark Matter (CDM).

Q41.
Answer: b
Explanation:
Recently, Indian Navy Chief unveiled the first indigenously manufactured Drishti 10 Starliner Unmanned Aerial
Vehicle (UAV).
● Drishti 10 Starliner Unmanned Aerial Vehicle (UAV) is an advanced Intelligence, Surveillance and
Reconnaissance (ISR) platform with 36 hours endurance and a 450 kg payload capacity. It is an all-
weather military platform which has clearance to fly in both segregated and unsegregated airspace.
● It is equipped with advanced communication systems, including satellite communication and Line-of-
Sight (LOS) data links, ensuring reliable and secure data transmission.
Therefore, option (b) is the correct answer.
Relevance: The Indian Navy Chief recently unveiled the first indigenously manufactured Drishti 10 Starliner.

Q42.
Answer: c
Explanation:
● Quantum gates are the basic building blocks of quantum circuits, like classical logic gates are for
conventional digital circuits. They are unitary matrices that operate on one or more qubits, changing
their quantum states. So, statement 1 is correct.
● Quantum gates are analogous to classical logic gates but operate on quantum bits or qubits. These gates
manipulate the quantum state of qubits, taking advantage of quantum phenomena such as
superposition and entanglement. They perform operations that transform the quantum state of the
qubits, allowing quantum computers to execute quantum algorithms. Common quantum gates include
Hadamard gates, Pauli gates (X, Y, Z), CNOT gates and others. So, statement 2 is correct.
● Quantum gates are reversible, meaning that the information about the initial state of the qubits can
be recovered from the final state. Unlike classical gates, quantum gates preserve the information and
do not result in a loss of data. So, statement 3 is correct.
○ Reversibility is a crucial property of quantum gates and is essential for the coherent evolution of
quantum states during computations.
● Applications of quantum gates and quantum computing include cryptography (breaking and creating
secure codes), optimization problems, simulation of quantum systems and machine learning.
So, all three of the above statements are correct.
24
Vajiram & Ravi PowerUp Current Affairs Test Series - 2024
CA Test No. – 10 (CA3320)
.
Therefore, option (c) is the correct answer.
Knowledge Box
Key terms:
● Superposition:
○ It is the quantum phenomenon where a quantum system, such as a particle or
a wave, can exist in multiple states or places at the exact same time.
○ For example, a photon can be both horizontally and vertically polarized, or an
electron can be both spin up and spin down, until they are measured and
collapse to one definite state.
○ Superposition allows quantum systems to explore multiple possibilities
simultaneously, which can lead to faster and more efficient computation and
communication.
● Entanglement:
○ It is the quantum phenomenon where two or more quantum systems, such as
particles or waves, become so strongly correlated that they cannot be
described independently of each other, even when they are separated by large
distances.
○ For example, two photons can be entangled such that measuring the
polarization of one photon will instantly reveal the polarization of the other
photon, regardless of how far apart they are.
○ Entanglement creates a special connection between quantum systems that can
enable secure encryption, teleportation and distributed computation

Q43.
Answer: c
Explanation:
● The Green Fuels Alliance India (GFAI) is a strategic initiative between India and Denmark to collaborate
in the sustainable energy sector and to achieve carbon neutrality.
● It is led by the Danish Embassy and the Consulate General of Denmark in India and involves nine Danish
organisations and several Indian partners. The GFAI aims to advance the green fuels sector, including
green hydrogen, through innovation and partnerships. It was announced in January 2024, as part of the
Green Strategic Partnership signed in 2020 between the two countries.
● Its primary objective is to promote sustainable energy growth in India by establishing an ecosystem that
encourages collaboration among businesses, government entities, research institutions and financial
stakeholders.
Therefore, option (c) is the correct answer.
Knowledge Box

Carbon Neutrality:

25
Vajiram & Ravi PowerUp Current Affairs Test Series - 2024
CA Test No. – 10 (CA3320)
.
● It is reached when the same amount of carbon dioxide (CO2) is released into the
atmosphere as is removed by various means, leaving a zero balance, also known as
a zero-carbon footprint.
Net Zero:
● It refers to a state in which the greenhouse gases going into the atmosphere are
balanced by removal out of the atmosphere.
● It is similar in principle to carbon neutrality, but is expanded in scale. To achieve net
zero means to go beyond the removal of just carbon emissions towards all Green
House Gases
Relevance: At the recent Global Investors Meet (GIM) 2024, Denmark announced a Green Fuels Alliance India
(GFAI) initiative.

Q44.
Answer: d
Explanation:
● The Great Oxidation Event (GOE) was a period when the Earth's atmosphere and shallow oceans first
experienced a rise in oxygen concentration. The GOE is also known as the Oxygen Catastrophe, Oxygen
Revolution, Oxygen Crisis or Oxygen Holocaust.
● The GOE is estimated to have occurred between 2.5 and 2.3 billion years ago.However, it's not known
whether the change was abrupt and permanent or gradual and drawn out over millions of years.
● It was caused by the photosynthesis of cyanobacteria, which produced oxygen as a by-product.
● It had major consequences for the evolution of life, as it enabled the emergence of aerobic organisms
and multicellular forms, but also caused the extinction of many anaerobic species that could not tolerate
oxygen.
Therefore, option (d) is the correct answer.
Relevance: A recent study has revealed the oldest evidence of photosynthesis, the same period of the Great
Oxidation Event.

Q45.
Answer: b
Explanation:
● Lentil (Masoor) is a cool-season crop that requires cold temperature during its vegetative growth and
warm temperature at the time of maturity. The optimum temperature for growth is 18-30°C. So,
statement 1 is not correct.
● Lentil requires a cold climate. It is very hardy and can tolerate frost and severe winters to a great extent.
Well drained, loam soils with neutral reaction are best for lentil cultivation. Acidic soils are not fit for
growing lentils.
● India is set to become the world’s largest producer of lentil (masoor) during the 2023-24 crop year on
account of higher acreage. Despite being the second largest producer of lentils, India has so far been
relying on imports to meet its domestic requirements, buying mainly from Australia, Canada, Russia,
Singapore and Turkey. So, statement 2 is correct.

26
Vajiram & Ravi PowerUp Current Affairs Test Series - 2024
CA Test No. – 10 (CA3320)
.
● The Government of India has increased the Minimum Support Price (MSP) of lentils for the marketing
season 2023-24 to Rs 6,000 per quintal, which is the highest increase among all rabi crops. In 2023-24,
the Government had increased the MSP of lentils to Rs 6,425 per quintal for the Rabi Marketing Season
2024-25. This is expected to ensure remunerative prices to the farmers and incentivize crop
diversification. So, statement 3 is correct.
So, only two of the above statements are correct.
Therefore, option (b) is the correct answer.
Relevance: India is on track to become the world’s largest producer of lentil (masoor) in the 2023-24 crop year.

Q46.
Answer: d
Explanation:
● The source of fluoride contamination in groundwater is either anthropogenic or geogenic.
Anthropogenic sources such as brick industry, aluminium smelters and the use of coal burning,
industrial activities and phosphate fertilisers significantly raise the groundwater fluoride level. So, points
1, 2, 3 and 4 are correct.
● However, the geogenic source includes the leaching and dissolution of fluoride-bearing minerals, ion
exchange, rock-water interaction, weathering process, evaporation, precipitation of calcite and type of
rock present.
● Prosopis juliflora is a hyper-accumulator of Fluoride and has the potential to remediate fluoride-
contaminated soils. Fluoride ion concentration in drinking water can be easily detected by ion-selective
electrodes. Various defluoridation techniques have been developed to reduce the fluoride content to
the desired level including principally membrane and adsorption processes.
● Fluoride normally enters the human body through water, food, industrial exposure, drugs, cosmetics
etc. The fluoride levels of food depend upon the nature of the soil and the quality of water used for
irrigation and thus vary from place to place.
So, all four of the above are the sources of fluoride contamination in groundwater.
Therefore, option (d) is the correct answer.
Relevance: The Central Ground Water Board (CGWB) recently shared the data on arsenic and fluoride
contamination with the Haryana Water Resources Authority (HWRA).

Q47.
Answer: c
Explanation:
● Cosmic rays are high-energy particles, primarily originating from sources outside our solar system.
These cosmic rays originate from outside our solar system and are believed to come from various
astrophysical sources such as supernovae, pulsars and other high-energy events in the Milky Way and
beyond. So, statement 1 is correct.
● Cosmic rays consist of charged particles like protons and heavier atomic nuclei. They are mostly ionised
atoms that have lost some or all of their electrons due to the intense radiation in space. The most
abundant cosmic rays are protons, which are the nuclei of hydrogen atoms, followed by alpha particles,

27
Vajiram & Ravi PowerUp Current Affairs Test Series - 2024
CA Test No. – 10 (CA3320)
.
which are the nuclei of helium atoms. There are also some heavier nuclei, such as carbon, oxygen, iron,
or even uranium, but they are much less common. So, statement 2 is correct.
○ Cosmic rays also include some electrons, which are negatively charged particles, but they are
only about 1% of the total.
● Only low-intensity cosmic rays reach the Earth’s surface, as most of the high-intensity ones are blocked
by the Earth’s magnetic field and atmosphere. The Earth’s magnetic field deflects the charged particles
away from the poles, where the field lines are vertical and towards the equator, where they are
horizontal. So, statement 3 is correct.
○ The atmosphere acts as a shield that absorbs the energy of the cosmic rays and produces
secondary particles, such as neutrons, muons, pions, or gamma rays.
○ These secondary particles can reach the ground, but they are much less energetic than the
primary cosmic rays.
So, only three of the above statements are correct.
Therefore, option (c) is the correct answer.
Knowledge Box

● The Amaterasu particle was the second-highest-energy cosmic ray ever detected, but
one of the highest. The highest-energy cosmic ray ever detected was the Oh-My-God
particle, which was observed in 1991. The Amaterasu particle was detected in 2021 and
later identified in 2023.
Relevance: A cosmic ray that has been dubbed ‘Amaterasu’ appears to have reached the earth from an empty
part of the universe.

Q48.
Answer: c
Explanation:
● ‘Tricholime’ is a granular lime-based formulation, integrating Trichoderma, a fungal biocontrol agent
used for controlling a variety of soil-borne pathogens and lime into a single product. This makes the
application easier for the farmers.
● Tricholime neutralises the soil acidity while promoting plant growth and shields crops from soil-borne
pathogens. It can also boost the growth of useful microbes in soil and also benefits the crop by improving
the physical condition of the soil, enhancing secondary nutrient availability and by boosting soil microbial
activity.
Therefore, option (c) is the correct answer.
Relevance: The Indian Institute of Spices Research (IISR) Kozhikode has successfully developed a new granular
lime-based Trichoderma formulation, ‘Tricholime’.

Q49.
Answer: a
Explanation:
● X-rays are a form of electromagnetic radiation similar to visible light. Unlike light, x-rays have higher
energy and can pass through most objects, including the body. Medical x-rays are used to generate
28
Vajiram & Ravi PowerUp Current Affairs Test Series - 2024
CA Test No. – 10 (CA3320)
.
images of tissues and structures inside the body. If x-rays travelling through the body also pass through
an x-ray detector on the other side of the patient, an image will be formed that represents the
“shadows” formed by the objects inside of the body. So, statement 1 is correct.
● Stars like the Sun mostly emit unpolarised light, although there exist space objects like Pulsars with
great magnetic fields. Electrons in those objects spiral around the magnetic field lines and emit X-rays.
The gyrating motion of such electrons provides the X-rays with some directionality. If the magnetic
field lines are mostly aligned in some direction, then the electrons would twirl in the perpendicular
direction. This would make the resulting X-rays polarised in the same direction. Measuring the
polarisation of X-rays would then enable astronomers to gauge the directions of magnetic fields in
these objects. So, statement 2 is correct.
● Earth's atmosphere absorbs most X-rays and gamma-rays. Earth's atmosphere is thick enough that
virtually none are able to penetrate from outer space all the way to the Earth's surface. Also, their high
energy makes it impossible to focus with lenses, like ordinary light. So, statement 3 is not correct.
Therefore, option (a) is the correct answer.
Relevance: Recently, the X-ray Polarimeter Satellite (XPoSat) of the Indian Space Research Organisation (ISRO)
was put into orbit for astronomical research.

Q50.
Answer: b
Explanation:
● The Himalayan Wolf (Canis lupus chanco) is a prominent lupine predator found across the Himalayas.
It has been assessed for the first time in the International Union for Conservation of Nature (IUCN)’s Red
List of Threatened Species. It has been categorised as ‘Vulnerable’ on the IUCN Red List. The assessment
added that all individuals were in one subpopulation stretching across the Himalayan range of Nepal
and India and the Tibetan Plateau. So, statement 1 is not correct and statement 2 is correct.
● It highlighted that hybridisation with dogs was an emerging threat to the Himalayan wolf population in
Ladakh and Spiti where increasing populations of feral dogs pose a growing challenge. The wolf is also
illegally hunted for trade in its fur and body parts including paws, tongues, heads and other parts. So,
statement 3 is correct.
● India is also home to the Indian/Common/Peninsular Wolf (Canis lupus pallipes) which is found in the
plains and the Deccan Plateau.
Therefore, option (b) is the correct answer.
Relevance: The Himalayan Wolf has been assessed for the first time in the International Union for Conservation
of Nature (IUCN)’s Red List of Threatened Species.

Q51.
Answer: c
Explanation:

29
Vajiram & Ravi PowerUp Current Affairs Test Series - 2024
CA Test No. – 10 (CA3320)
.
● A Light Emitting Diode (LED) is a diode that emits light. Inside the diode’s p-n junction, the electrons
have more energy than the holes. When an electron meets and occupies a hole, it releases energy into
its surroundings. If the frequency of this energy is in the visible part of the electromagnetic spectrum,
the diode will be seen to emit light. The overall phenomenon is called electroluminescence. So,
statement 1 is correct.
● LEDs can produce all three primary colours - red, green and blue. Scientists were able to create red and
green LEDs for more than 40 years before they created blue LEDs. So, statement 2 is correct.
● LEDs are “directional” light sources, which means they emit light in a specific direction, unlike
incandescent and Compact Fluorescent Lighting (CFL), which emits light and heat in all directions. So,
statement 3 is correct.
So, all three of the above statements are correct.
Therefore, option (c) is the correct answer.
Knowledge box

Advantages of LEDs:
● They are more efficient than incandescent and fluorescent bulbs.
● They have several applications in industry, consumer electronics and household
appliances: from smartphones to TV screens, from signboards to ‘feeding’ plants light
in greenhouses, from barcode scanners to monitoring air quality.
Relevance: New research has thrown light on LEDs being more efficient than incandescent bulbs and fluorescent
lamps.

Q52.
Answer: a
Explanation:
● The National Essential Diagnostics List (NEDL) lists the essential and most basic tests that should be
available at various levels of healthcare facilities in the country, including at the village level, in sub-
health centres, health and wellness centres and primary health centres.
● The development and implementation of an NEDL was recommended by the World Health Organisation
(WHO) to facilitate the availability of in-vitro diagnostics across the various tiers of the healthcare
pyramid.
○ India’s first NEDL was released by the Indian Council of Medical Research (ICMR) in 2019 to
make the availability of diagnostics an essential component of the healthcare system.
● The ICMR defined essential diagnostic tests as those that satisfy the priority healthcare needs of the
population and are selected with due regard to disease prevalence and public health relevance, evidence
of efficacy and accuracy and comparative cost-effectiveness.
Therefore, option (a) is the correct answer.
Relevance: The Indian Council of Medical Research (ICMR) has begun the process of revising the current
National Diagnostics List (NEDL).

Q53.
Answer: a
30
Vajiram & Ravi PowerUp Current Affairs Test Series - 2024
CA Test No. – 10 (CA3320)
.
Explanation:
● A superconductor is a material that achieves superconductivity, which is a state of matter that has no
electrical resistance and does not allow magnetic fields to penetrate. An electric current in a
superconductor can persist indefinitely as there is no electrical resistance. So, statements 1 and 2 are
correct.
● Superconductivity can only typically be achieved at very cold temperatures. In 1957, physicists John
Bardeen, Leon N. Cooper and Robert Schrieffer developed the theory of superconductivity. To create
electrical resistance, the electrons in a metal need to be free to bounce around. But when the electrons
inside a metal become incredibly cold, they can pair up, preventing them from bouncing around. These
electron pairs, called Cooper pairs, are very stable at low temperatures, and with no electrons "free" to
bounce around, the electrical resistance disappears.
● Superconductors have a wide variety of everyday applications, from Magnetic Resonance Imaging
machines to super-fast maglev trains that use magnets to levitate the trains off the track to reduce
friction. Researchers are now trying to find and develop superconductors that work at higher
temperatures, which would revolutionize energy transport and storage.
So, both Statement-I and Statement-II are correct and Statement-II is the correct explanation for Statement
I.
Therefore, option (a) is the correct answer.
Relevance: Recently two South Korean researchers claimed on the internet that a lead-based compound they
had developed had shown superconducting properties at room temperature.

Q54.
Answer: a
Explanation:
● The Crystal Awards are announced annually by the World Economic Forum (WEF). The award is
presented at Davos each year by the chairperson of the World Economic Forum (WEF). So, statement 1
is not correct.
● These awards celebrate the achievements of leading artists who are bridge-builders and role models
for all leaders of society. So, statement 2 is correct.
● Several Indian artists have received the Crystal Award in the past including actor Amitabh Bachchan,
musician A.R. Rahman, sarod player Amjad Ali Khan, sitarist Ravi Shankar, artist Mallika Sarabhai and
actresses Shabana Azmi and Deepika Padukone. So, statement 3 is not correct.
So, only one of the above statements is correct.
Therefore, option (a) is the correct answer.
Relevance: Recently, the World Economic Forum announced the 2024 Crystal Awards.

Q55.
Answer: a
Explanation:
● The Punganur is an indigenous cow breed native to Andhra Pradesh. They are a unique dwarf breed
with the shortest hump in the world.
● The Sanchori are indigenous cattle of Rajasthan. They are good milk producers.

31
Vajiram & Ravi PowerUp Current Affairs Test Series - 2024
CA Test No. – 10 (CA3320)
.
● Masilum cattle are well adapted to the hill ecosystem of Meghalaya. The Khasi language has the words
‘Masi’ and “Lum” which means cattle and hills, so it is called “Masilum”. These indigenous cattle are
reared by the Khasi and Janitia communities.
Therefore, option (a) is the correct answer.
Relevance: The Prime Minister of India on the occasion of Makar Sankranti fed the Punganur breed of cows at
his residence in New Delhi.

Q56
Answer: c
Explanation:
● Vaccine Safety Net (VSN) is a global network of websites, established by the World Health Organization
(WHO) that provides reliable information on vaccine safety. So, statements 1 and 2 are correct.
● It is a network of a diverse group of digital information resources (websites and social media), VSN
members, located in countries around the world and providing scientifically based information on
vaccine safety in various languages.
● A key player in the Project is the Global Advisory Committee on Vaccine Safety (GACVS), established by
WHO in 1999, to respond promptly, efficiently, and with scientific rigour to vaccine safety issues of
potential global importance.
● It is continuously expanding and to date, 110 websites from 45 countries provide vaccine safety
information in 43 languages.
Therefore, option (c) is the correct answer.
Relevance: The Healthy Indian Project (THIP), a health information platform in India, is included as a member
of the World Health Organization’s Vaccine Safety Net (VSN).

Q57.
Answer: b
Explanation:
● METSAT, which was renamed as ‘Kalpana-1’ is the first exclusive meteorological satellite built by Indian
Space & Research Organisation (ISRO). It is placed in a geosynchronous transfer orbit. It served as a
precursor to the INSAT system of satellites. So, point 1 is correct.
● INSAT-3D is an advanced weather satellite of India configured with an improved Imaging System and
Atmospheric Sounder. It is designed for enhanced meteorological observations, monitoring of land and
ocean surfaces and generating a vertical profile of the atmosphere in terms of temperature and humidity
for weather forecasting and disaster warning. It is placed in a geostationary orbit. So, point 2 is correct.
● CARTOSAT-3 is a third-generation, agile and advanced satellite, with a very high-resolution imaging
capability. It was launched to meet the increasing user demands for cartographic applications. It has
applications in urban and rural resource management, coastal land use, utility mapping and GIS
applications. It was placed in Sun Synchronous Polar orbit. So, point 3 is not correct.
● SCATSAT-1 is a continuity mission for the Oceansat-2 Scatterometer dedicated to ocean wind
observation. Major objectives of SCATSAT-1 are weather forecasting and cyclone prediction along with
ocean state monitoring and prediction. It was placed in the Polar Sun Synchronous Orbit. So, point 4 is
correct.

32
Vajiram & Ravi PowerUp Current Affairs Test Series - 2024
CA Test No. – 10 (CA3320)
.
● The GSAT-7B is a communication satellite part of the GSAT-7 series, which was first launched by the
ISRO. It primarily fulfils the communication needs of the Army. So, point 5 is not correct.
Therefore, option (b) is the correct answer.
Relevance: The Kalpana 1 and INSATs 3A, 3D and 3DR satellites have bolstered India’s weather monitoring and
warning services.

Q58.
Answer: c
Explanation:
● When a pure water sample is cooled, it can remain in the liquid state at temperatures well below its
melting point (0°C). The initiation of the transition from the liquid state to ice is called nucleation.
Substances that facilitate this transition so that it takes place at a relatively high sub-zero temperature
are called Ice Nucleators.
● Water molecules need an initiation point or a nucleus to correctly orient themselves to create a
crystalline structure. This nucleus could be an ice particle or an impurity like dust, minerals, or
microorganisms commonly found in water. All the above particles in ice serve as Ice Nucleators by
triggering the nucleation process.
Therefore, option (c) is the correct answer.
Knowledge Box

Biological Ice Nucleators:


● Several microorganisms like bacteria, lichen and fungi have evolved to trigger the
nucleation process in water so that it forms ice more easily. This phenomenon is called
biological ice nucleation and the organisms are called biological ice nucleators.
● Bacteria like Pseudomonas syringae act as biological ice nucleators by producing
special ice nucleation proteins (INPs) near their cell membranes which become anchor
points for water molecules to start forming ice crystals.
● According to recent studies, Fungi act more efficiently than bacteria in the initiation of
nucleation. It is enabled through extracellular ice nucleation proteins produced by
fungi.
Relevance: In a recent study, scientists have found that fungi can act as better ice nucleators than bacteria.

Q59.
Answer: c
Explanation:
● The International Court of Justice (ICJ) is the highest judicial body of the United Nations (UN). It was
established in 1945 by the charter of the United Nations, following the two World Wars. It is seated at
the Peace Palace in The Hague, the Netherlands. So, statement 1 is correct.

33
Vajiram & Ravi PowerUp Current Affairs Test Series - 2024
CA Test No. – 10 (CA3320)
.
○ The court has a total strength of 15 judges who are appointed for nine-year terms through
separate, simultaneous elections at the UN General Assembly and the UN Security Council.
However, ‘ad-hoc judges’ are sometimes appointed in cases that require specialised knowledge
about the concerned dispute. ICJ judges ought to be impartial and cannot act as extensions of
their countries.
● Only States are eligible to appear before the Court in contentious cases. At present, this essentially
means the 193 Member States of the United Nations. The Court has no jurisdiction to deal with
applications from individuals, non-governmental organizations, corporations or any other private
entity. It cannot provide them with legal advice or help them in their dealings with national authorities.
However, a State may take up the case of one of its nationals and invoke against another State the
wrongs which its national claims to have suffered at the hands of the latter; the dispute then becomes
one between States. So, statement 2 is correct.
● The ICJ has no jurisdiction to try individuals accused of war crimes or crimes against humanity. As it is
not a criminal court, it does not have a prosecutor able to initiate proceedings. So, statement 3 is correct.
● The Court’s decisions are legally binding. The UN charter authorises the United Nations Security Council
to enforce the court’s decisions but its compliance is often at the mercy of the power politics of the
permanent members of the Security Council.
So, all three of the above statements are correct.
Therefore, option (c) is the correct answer.
Relevance: The International Court of Justice recently heard the case instituted by South Africa accusing Israel
of committing genocide in Gaza.

Q60.
Answer: c
Explanation:
● Thorium is commonly found in igneous rocks and heavy mineral sands. It is three times more abundant
in nature than uranium but historically has found little use in industry or power generation. This is
because thorium itself is not a nuclear fuel, although it can be used to create such a fuel. So, statement
1 is correct.
● Thorium-232, the only naturally occurring isotope of thorium, is considered ‘fertile’ for fission. This
means that it needs a driver, such as uranium or plutonium, to trigger and maintain a chain reaction.
When irradiated, thorium-232 undergoes a series of nuclear reactions, eventually forming uranium-233,
which can then be split to release energy to power a nuclear reactor. So, statement 2 is correct.
● Thorium is obtained from monazite and ilmenite. India has the largest reserves of monazite and
ilmenite in the world. It is found in the coastal sand of Kerala, where it is centralized mostly in the
districts of Kollam and Palakkad. The Hills of Travancore is its source. Ilmenite is found mainly in the
coastal sand from Ratnagiri, that is, Konkan, to Kerala. It is also found in the coastal sand of Tamil Nādu
and Odisha. So, statement 3 is correct.
So, all three of the above statements are correct.
Therefore, option (c) is the correct answer.
Relevance: Recent developments at the Bhabha Atomic Research Centre (BARC) give hope that thorium
minerals can be India’s answer to green energy.
34
Vajiram & Ravi PowerUp Current Affairs Test Series - 2024
CA Test No. – 10 (CA3320)
.
Q61.
Answer: b
Explanation:
● Scientists have created a new breed of robots called self-growing robots that draw inspiration from the
remarkable adaptive behaviours of climbing plants like responding to gravity and light. These
autonomous growing robots possess the ability to navigate and explore unstructured environments,
paving the way for unprecedented advancements in the field of soft robotics. So, statement 1 is correct.
● These robots can autonomously construct complex three-dimensional models through the
implementation of additive manufacturing techniques. This results in a more reliable structure with
fewer assembled components and enhanced biomimetic behaviours. So, statement 2 is correct.
● They mimic the growth pattern seen in climbing plants and adaptively add material to construct their
bodies. This unique characteristic allows them to navigate above and below-ground environments,
penetrate dense media like soil and deftly negotiate unpredictable obstacles. So, statement 3 is not
correct.
So, only two of the above statements are correct.
Therefore, option (b) is the correct answer.
Relevance: Recently, scientists have created a self-growing robot FiloBot that navigates like climbing plants.

Q62.
Answer: a
Explanation:
● Lakshadweep is an archipelago consisting of 36
islands with an area of 32 sq km. It is a uni-district
Union Territory and comprises 12 atolls, three reefs,
five submerged banks and ten inhabited islands. The
capital is Kavaratti and it is also the principal town of
the UT. All Islands are 220 to 440 km away from the
coastal city of Kochi in Kerala, in the Arabian Sea.
● The Eight degree channel separates Lakshadweep
from Maldives. Through this channel pass the Sea
lines of Communication along which flows the heavy
East-West maritime traffic that keeps the wheels of
the global economy turning.
Therefore, option (a) is the correct answer.
Relevance: The relations between India and the Maldives
soured since the change of regime in the island nation.

Q63.
Answer: c
Explanation:

35
Vajiram & Ravi PowerUp Current Affairs Test Series - 2024
CA Test No. – 10 (CA3320)
.
● A soda lake is an alkaline lake with a pH value of more than the usual measure of 6 or 7, usually between
9 and 11. Soda lakes are mostly found within endorheic (closed) basins and their occurrence depends
on the combination of climatic, geological and geochemical factors. For the formation of soda lakes, a
topography that limits the outflow of water from the lake is needed and an endorheic basin provides
that. So, statements 1 and 3 are correct.
○ Examples of soda lakes are Mono Lake in the USA, Lake Van in Turkey, Lonar Lake in India etc.
● Soda lakes are inhabited by a rich diversity of microbial life making them a productive ecosystem. Soda
lakes also harbour unique species which are adapted to the alkali conditions. These organisms which are
adapted to the high alkalinity are called haloalkaliphiles. So, statement 2 is correct.
● Soda lakes get their name from having high levels of dissolved sodium and carbonate. They can also
have high levels of dissolved phosphate. As per researchers, these lakes have 50,000 times more
phosphorus than is found in seawater, rivers and other types of lakes.
So, all three of the above statements are correct.
Therefore, option (c) is the correct answer.
Relevance: Recently, studies highlighted that shallow soda lakes hold promise as cradles of life on Earth.

Q64.
Answer: a
Explanation:
● The Sulfur Fluoride Exchange (SuFEx) is a promising new click reaction that rests on the elevated
reactivity of sulfonyl fluorides and fluorosulfates.
● Recently, the SuFEx chemistry has been used to create Sulfoximines, sulfonimidoyl fluorides and
sulfonimidamides, which are types of sulfur-containing chemical compounds that have wide-ranging
potential as therapeutic drugs. However, the synthesis process for these compounds is complex and has
several limitations, such as the requirement for the use of high pressures.
● SuFEx click chemistry has various important advantages over other click reactions in that it is oxygen and
water-friendly, ultraviolet and thermally inert. This peculiar reactivity of SuFEx has drawn major interest
for its easy manipulation, high yields, fast reaction rates and high tolerance towards various functional
groups.
Therefore, option (a) is the correct answer.
Knowledge Box

Click chemistry:
● It is a set of powerful, highly reliable and selective reactions for the rapid synthesis of
useful new compounds and combinatorial libraries through heteroatom links. In simple
terms, it is a functional field where molecules snap together quickly and efficiently –
literally like a click.
● Because of their high selectivity and specificity, click chemistry reactions find wide-
range applications in areas like drug conjugation, materials science, etc.

36
Vajiram & Ravi PowerUp Current Affairs Test Series - 2024
CA Test No. – 10 (CA3320)
.
● The 2022 Nobel Prize in Chemistry has been awarded to chemists Carolyn R. Bertozzi
and K. Barry Sharpless from the U.S. and Morten Meldal from Denmark, for their work
in the field of click chemistry and bioorthogonal chemistry.
Relevance: Recent studies highlighted that the Sulfur Fluorine Exchange (SuFEx) improves the process of making
Sulfur-containing compounds that may be used in medicines.

Q65.
Answer: a
Explanation:
• Raptors, such as eagles, hawks, and falcons, are apex predators as they are at the top of their food
chain, preying on smaller animals. As scavengers, they help to recycle the nutrients back into soil and
prevent the spread of diseases around carcasses. So, statement 1 is correct.
• Raptors do not have gestation periods because they lay eggs. Instead of carrying developing embryos
internally, like mammals, they lay eggs and incubate them until they hatch. The incubation period varies
among species. Raptors typically build their own nests and lay eggs in them. They invest time and effort
in selecting and constructing nests in high locations, such as trees or cliffs, to ensure the safety of their
offspring. So, statement 2 is not correct.
o All birds incubate their eggs, except megapodes (mound builders), which depend on the heat
generated by decaying vegetation or other external sources, and brood parasites such as cuckoos
and cowbirds, which lay their eggs in the nests of other species.
• Many raptors are known for their wide-ranging habitats. They can be found in diverse environments,
including forests, grasslands, deserts and mountainous regions. Their ability to cover extensive areas is
crucial for hunting and finding suitable nesting sites. So, statement 3 is not correct.
So, only one of the above statements is correct.
Therefore, option (a) is the correct answer.
Relevance: New research reveals that 88 per cent of African raptor species have declined over the past 20 to 40
years.

Q66.
Answer: b
Explanation:
● Seamounts are underwater mountains that rise hundreds or thousands of feet from the seafloor but do
not reach the surface of the water. They are typically conical or dome-shaped, with steep slopes and
rugged terrain. Like volcanoes on land, seamounts can be active, extinct or dormant volcanoes. They are
formed through volcanic activity. So, statement 1 is correct.
● Seamounts are recognised as hotspots for marine life. They are home to diverse biological
communities. They are good places for life because they can cause localised ocean upwelling – the
process by which nutrient-rich water from deep within the ocean moves up to the surface. So, statement
2 is correct.
● The highest mountain on Earth is a seamount, Hawaii’s Mauna Kea, a dormant volcano. It is more than
30,000 feet tall measured from its base on the seafloor 18,000 feet beneath the surface. So, statement
3 is not correct.
So, only two of the above statements are correct.

37
Vajiram & Ravi PowerUp Current Affairs Test Series - 2024
CA Test No. – 10 (CA3320)
.
Therefore, option (b) is the correct answer.
Knowledge Box
● Most seamounts are formed near mid-ocean ridges, where the earth’s tectonic
plates are moving apart, allowing molten rock to rise to the seafloor. Some
seamounts have also been found near intraplate hotspots, regions of heavy
volcanic activity within a plate and oceanic island chains with volcanic and seismic
activity called island arcs.
● Seamounts are formed when molten rock comes up from below the tectonic
plates. They provide information about the mantle’s composition and about how
tectonic plates evolve. They are generally extinct volcanoes that, while active,
create piles of lava that sometimes break the ocean surface.
Relevance: In a recent discovery, scientists have reported finding 19,325 new seamounts after poring through
new high-resolution data.

Q67.
Answer: b
Explanation:
● The Design Linked Incentive (DLI) scheme was launched by the Ministry of Electronics and Information
Technology (MeitY) to offset the disabilities in the domestic industry involved in semiconductor design.
It aims to not only move up in the value chain but also strengthen the semiconductor chip design
ecosystem in the country. So, statement 1 is correct.
● Financial incentives and design infrastructure support will be extended to domestic companies, Startups
and MSMEs engaged in semiconductor design or semiconductor-linked design.
● The approved applicants that claim incentives under the scheme shall retain their domestic status (i.e.
more than 50% of the capital in it is beneficially owned by resident Indian citizens and/ or Indian
companies, which are ultimately owned and controlled by resident Indian citizens) for three years after
claiming incentives under the scheme. So, statement 2 is correct.
● Centre for Development of Advanced Computing (C-DAC), a scientific society operating under MeitY,
will serve as the nodal agency for the implementation of the DLI scheme. So, statement 3 is not correct.
So, only two of the above statements are correct.
Therefore, option (b) is the correct answer.
Relevance: Since its announcement, the DLI scheme has approved only seven start-ups, markedly short of its
target of supporting 100 over five years.

Q68.
Answer: b
Explanation:
● Organoids are three‐dimensional (3D) miniaturized versions of organs or tissues that are derived from
cells with stem potential. They can self‐organize and differentiate into 3D cell masses, recapitulating the
morphology and functions of their in vivo counterparts. So, statement 1 is correct.

38
Vajiram & Ravi PowerUp Current Affairs Test Series - 2024
CA Test No. – 10 (CA3320)
.
● They can be generated by inducing and culturing Pluripotent Stem Cells (iPSCs), Embryonic Stem Cells
(ESCs), Adult Stem Cells (ASCs) and tumour cells from healthy donors or patients. Organoids culture is
evolving and organoids derived from various organs and tissues, such as the brain, lung, heart, liver and
kidney have been generated. So, statement 2 is not correct.
● With retinal organoids, researchers, recently, discovered how an offshoot of vitamin A generates the
specialised cells that enable people to see millions of colours, an ability that dogs, cats and other
mammals do not possess. The findings increase understanding of colour blindness, age-related vision
loss and other diseases linked to photoreceptor cells. So, statement 3 is correct.
○ The other potential uses of organoids include studying human biology and how diseases develop;
testing drugs for whether they will be safe and effective in a particular patient; and discovering
new pharmaceuticals.
So, only two of the above statements are correct.
Therefore, option (b) is the correct answer.
Relevance: With retinal organoids grown in a petri dish, researchers recently discovered how humans see
colours.

Q69.
Answer: c
Explanation:
● Probiotics are live bacteria found in certain foods or supplements whereas prebiotics are food for these
bacteria. Both prebiotics and probiotics are important for human health. So, statement 1 is correct.
○ Probiotics help humans from harmful bacteria and fungi. They can aid in immune system
functions, improve symptoms of depression and also help address obesity.
○ Prebiotics constitute the types of fiber that humans cannot digest but the gut bacteria can digest
them. These types of fiber provide nutrients to the bacteria that support healthy digestion and
immune function.
● Probiotic foods that naturally contain helpful bacteria include yogurt, fermented foods etc. Prebiotics
are found in fiber-rich fruits, vegetables and legumes. Synbiotic foods are those that contain both
beneficial bacteria and a prebiotic source of fiber. Cheese is an example of synbiotic food. So, statement
2 is correct.
Therefore, option (c) is the correct answer.
Relevance: New research demonstrates how probiotics promote weight loss in obese dogs.

Q70.
Answer: a
Explanation:
● Encryption is the act of changing some consumable information into an unconsumable form based on
some rules. There are different kinds of such rules.
● End-to-end (E2E) encryption refers to particular locations between which information moves. In E2E
encryption, the message is encrypted both in transit and at rest. It means that it is encrypted when
being relayed from one's phone to the server (or vice versa) and when it is sitting inside the server. It is
only decrypted when a person receives the message. So, statement 1 is not correct.

39
Vajiram & Ravi PowerUp Current Affairs Test Series - 2024
CA Test No. – 10 (CA3320)
.
● There are several ways to encrypt information depending on the level of secrecy and protection
required. Major types include symmetric and asymmetric encryption. In symmetric encryption, the key
used to encrypt some information is also the key required to decrypt it and in asymmetric encryption,
the key used to encrypt is different from the key required to decrypt. So, statement 2 is not correct.
● Symmetric encryption is useful when the sender and the recipient are the same person. The Advanced
Encryption Standard (AES), which is noticed when setting one's WiFi password, is a symmetric
encryption algorithm. So, statement 3 is correct.
So, only one of the above statements is correct.
Therefore, option (a) is the correct answer.
Relevance: Recently, Meta has added end-to-end encryption to Facebook and Messenger.

Q71.
Answer: c
Explanation:
● A taxpayer is a person who either has filed a return of income for the relevant Assessment Year (AY) or
in whose case tax has been deducted at source in the relevant financial year but the taxpayer has not
filed the return of income.
● As per the recent data released by the Central Board of Direct Taxes (CBDT), in the overall tax collections
by the government, direct tax collections accounted for 54.62 per cent share in the Financial Year (FY)
2022-23, a four-year high. Direct taxes as a share of the overall taxes had stood at 52.27 per cent in FY22,
46.84 per cent in FY21, 52.42 per cent in FY20 and 54.83 per cent in FY19. So, statement 1 is correct.
● The direct tax-to-GDP ratio, which reflects the share of taxes in the overall output generated in the
country, rose to a 15-year high of 6.11 per cent in the financial year 2022-23. This was also accompanied
by an increase in the income tax return filers in India to 7.4 crore in FY23, up 6.3 per cent from FY22. As
the economy grows and per capita income increases, the tax-to-GDP ratio also increases. So, statement
2 is correct.
● A tax-to-GDP ratio is a gauge of a nation's tax revenue relative to the size of its economy as measured
by GDP. The ratio provides a useful look at a country's tax revenue because it reveals potential taxation
relative to the economy. Higher tax revenues mean a country can spend more on improving
infrastructure, health and education which are keys to the long-term prospects for a country's economy
and people.
Therefore, option (c) is the correct answer.
Relevance: The direct tax-GDP ratio rose to a 15-year high in the Financial Year 2022-2023.

Q72.
Answer: a
Explanation:
• The Global Goal on Adaptation (GGA) was established under the Paris Agreement to enhance climate
change adaptation by increasing awareness of and funding towards countries’ adaptation needs in the
context of the 1.5/2°C goal of the Paris Agreement.
• However, unlike the clear 1.5°C target for global mitigation, adaptation is primarily a local activity and
as such, a global target has been challenging to establish.

40
Vajiram & Ravi PowerUp Current Affairs Test Series - 2024
CA Test No. – 10 (CA3320)
.
• At COP26, Parties established the 2022-2023 Glasgow-Sharm el-Sheikh (GlaSS) Work Programme on
the Global Goal on Adaptation to define the key elements of the goal, including the methodologies,
indicators, metrics, and data sources to support the assessment of overall adaptation progress.
Therefore, option (a) is the correct answer.

Q73.
Answer: c
Explanation:
● India and France have decided to set up the Indo-Pacific Triangular Cooperation (IPTDC) Fund. It aims to
support climate and SDG-focused innovations and Startups from third countries of the Indo-Pacific, to
facilitate the scaling up of green technologies being developed in the region. So, statements 1 and 2
are correct.
● Both countries will jointly identify the projects to be supported through the IPTDC Fund. This initiative
would be a significant step ahead in providing viable and transparent funding alternatives to innovators
in the Indo-Pacific region and would also be a key pillar of the India-EU Connectivity Partnership launched
in 2021.
Therefore, option (c) is the correct answer.
Relevance: Recently, India and France called for the early launching of the Indo-Pacific Triangular Development
Cooperation Fund.

Q74.
Answer: a
Explanation:
● NASA's Lunar Reconnaissance Orbiter (LRO) was focused on supporting the extension of human
presence in the solar system. It also helps to identify sites close to potential resources with high scientific
value, favourable terrain and the environment necessary for safe future robotic and human lunar
missions. The LRO mission has been extended to continue lunar science and exploration.
Therefore, option (a) is the correct answer.
Relevance: Recently, a Laser instrument onboard NASA's Lunar Reconnaissance Orbiter orbiting the Moon has
successfully pinged the Vikram lander of India's Chandrayaan-3 mission.

Q75.
Answer: a
Explanation:
● A private equity firm's strategy is to buy mostly mature companies that are already established. The
companies may be deteriorating or failing to make the profits they should due to inefficiency. Private
equity firms buy these companies and streamline operations to increase revenues. Venture capital firms,
on the other hand, mostly invest in startups with high growth potential. So, statement 1 is correct.
● Private equity firms can buy companies from any industry while venture capital firms tend to focus on
startups in technology, biotechnology and clean technology—although not necessarily. Private equity
firms also use both cash and debt in their investment, whereas venture capital firms deal with equity
only. These observations are common cases. However, there are exceptions to every rule; a firm may
act out of the norm compared to its competitors. So, statement 2 is not correct.

41
Vajiram & Ravi PowerUp Current Affairs Test Series - 2024
CA Test No. – 10 (CA3320)
.
Therefore, option (a) is the correct answer.
Relevance: Recently, private equity and venture capital investments in Indian companies declined.

Q76.
Answer: d
Explanation:
● While a Trust is governed by the Indian Trust Act, 1882, the Societies are governed by the Societies
Registration Act 1860. Many States, however, have variants of the Act. So, statement 1 is not correct.
● Minimum two trustees are required to register a public charitable Trust in general. Indian citizens serve
as trustees, although there is no prohibition against non-natural legal persons or foreigners serving in
this capacity. However, a minimum of 7 members are required for the formation of State level Society
and 8 members in case different States members are involved. So, statement 2 is not correct.
● Unlike Trusts, a Society has a more democratic set-up with membership and an elected body to manage
the society. Generally, a society works best for individuals with state-level objectives who are interested
in focusing on their formalities. So, statement 3 is not correct.
So, none of the above statements are correct.
Therefore, option (d) is the correct answer.

Q77.
Answer: b
Explanation:
● The Ice Memory Foundation aims to collect, save and manage ice cores from selected glaciers currently
in danger of degradation or disappearance, with their yielded information for decades and centuries to
come.
● Since the 1970’s, scientists have been carrying out ice core drilling operations for their own research. As
part of Ice Memory, six expeditions have been undertaken in an effort to collect heritage ice cores to
be stored in Antarctica. Once it is completely driven in – and thus filled with ice – glaciologists extract
the tube containing an ice cylinder of approximately 1.0 meter long and 10 cm. of diameter, known as
the ice core. So, statement 1 is not correct.
○ Each core is placed in a protective cover which is numbered and qualified in terms of its place of
extraction, the upper and lower end is identified as well as the depth from which it was extracted
and subsequently carefully stored in isothermal storage boxes until being transported in a
refrigerated container.
● Implementation of the Ice Memory initiative translates into different actions:
○ Ice core drilling of glaciers selected for their scientific value.
○ Analysing ice cores of reference to build an open-access database. So, statement 2 is correct.
○ Building a sanctuary in Antarctica dedicated to long-term storage of heritage ice cores.
○ Long-term governance
○ International mobilisation
Therefore, option (b) is the correct answer.
Relevance: Recently, as part of the Ice Memory initiative researchers analyzed ice cores drilled in 2018 and 2020
from the Corbassière glacier at Grand Combin in the canton of Valais.

42
Vajiram & Ravi PowerUp Current Affairs Test Series - 2024
CA Test No. – 10 (CA3320)
.
Q78.
Answer: d
Explanation:
● 'Multi-omics' refers to an approach in scientific research that involves the integration of various-omics
disciplines, such as genomics (study of genes), proteomics (study of proteins) and metabolomics (study
of metabolites).
● In recent times, research in precision oncology is moving towards a more comprehensive understanding
of tumours through multi-omics, which includes genomics, proteins and metabolites.
● At the same time, it is gaining wider application in identifying newer molecular subtypes of cancer with
implications for cancer progression and treatment.
Therefore, option (d) is the correct answer.
Relevance: Recently, UK research with 13,800 cancer patients showed that genomic integration with clinical
data can identify new cancer targets, guide treatment strategies and highlight inherited risks.

Q79.
Answer: a
Explanation:
The United Nations Educational, Scientific and Cultural Organization (UNESCO) seeks to encourage the
identification, protection and preservation of cultural and natural heritage around the world.
● China has the most World Heritage sites on the continent of Asia, with a staggering 55 locations. India
stands at second place in Asia and sixth in the world with 42 sites. Of these 34 are cultural, seven are
natural and one is of mixed heritage. So, statement 1 is not correct.
● Maharashtra has the highest number of UNESCO World Heritage sites currently. It has a total of six
heritage sites, out of which five are cultural sites and one is a natural heritage site. So, statement 2 is
not correct.
● Khangchendzonga National Park, located in Sikkim, is India's only Mixed Heritage Site. It is also included
in the World Network of Biosphere Reserves. It encompasses pristine forests, glacial lakes and the third-
highest peak in the world, Mount Khangchendzonga. It is a sanctuary for rare wildlife, including snow
leopards and red pandas. So, statement 3 is correct.
So, only one of the above statements is correct.
Therefore, option (a) is the correct answer.
Relevance: India has recently nominated 12 forts of Marathas for inclusion in the United Nations Educational,
Scientific and Cultural Organization (UNESCO) World Heritage list.

Q80.
Answer: b
Explanation:
● Range anxiety is the fear that an electric vehicle (EV) will not have enough battery charge to reach its
destination, leaving its occupants stranded. This anxiety is particularly prominent when considering
long-distance travel, along stretches of road where EV charging points might be few and far between.
● Range anxiety is a greater barrier to electrification in transportation than any of the other barriers, like
the cost and capability of batteries.
Therefore, option (b) is the correct answer.
43
Vajiram & Ravi PowerUp Current Affairs Test Series - 2024
CA Test No. – 10 (CA3320)
.
Relevance: Recently, Cornell University engineers have created a new lithium battery that can charge in under
five minutes faster than any such battery on the market.

Q81.
Answer: c
Explanation:
● A Green roof, or rooftop garden, is a vegetative layer grown on a rooftop. Green roofs offer benefits
like reduced air pollution and greenhouse gas emissions, reduced energy use, improved human health
and comfort, improved quality of life etc.
○ Green roof temperatures can be 30–40°F lower than those of conventional roofs and can reduce
city-wide ambient temperatures by up to 5°F. So, statement 3 is correct.
● Green roofs can reduce and slow stormwater runoff in the urban environment and also filter pollutants
from rainfall. Green roofs can retain nearly all storm-related precipitation during the summer months,
with lower retention during the winter months (< 20%). So, statements 1 and 2 are correct.
● Green roofs result in extended life expectancy of a roof. A green roof can be expected to experience
double or triple the life of a standard roof. This is possible because green roofs protect the
waterproofing membrane from wind-blown debris and ultraviolet radiation. They also provide
insulation from temperature extremes, thereby minimising damage from daily expansion and
contraction.
So, all three of the above statements are correct.
Therefore, option (c) is the correct answer.
Relevance: As per recent research, fungal-rich soil may improve green roof sustainability.

Q82.
Answer: c
Explanation:
● eROSITA (extended ROentgen Survey with an Imaging Telescope Array) is a wide-field X-ray telescope
on board the Russian-German "Spectrum-Roentgen-Gamma" (SRG) observatory. It was launched in 2019
and placed in a halo orbit around the L2 (Lagrange) point.
● It is a sensitive X-ray telescope capable of delivering deep, sharp images over very large areas of the
sky in the energy range of ~0.2-8 keV.
● Recently, the first eROSITA All-Sky Survey (eRASS1) was released which yielded the largest X-ray
catalogue ever published. It catalogs over half X-ray sky including supermassive black holes in distant
galaxies, X-ray emitting stars in our own Milky Way, clusters of galaxies, plus a small number of other
exotic classes of sources like X-ray-emitting binary stars, supernova remnants, pulsars and other
objects.
Therefore, option (c) is the correct answer.
Relevance: The first eROSITA sky-survey data release revealed the largest-ever catalogue of high-energy cosmic
sources.

Q83.
Answer: b
Explanation:
44
Vajiram & Ravi PowerUp Current Affairs Test Series - 2024
CA Test No. – 10 (CA3320)
.
A Ramsar site is a wetland site designated to be of international importance under the Ramsar Convention, also
known as "The Convention on Wetlands". It is an intergovernmental environmental treaty established in 1971
by the United Nations Educational, Scientific and Cultural Organization (UNESCO), which came into force in
1975.
● Sirpur Wetland: The site is a human-made wetland situated in Madhya Pradesh. It is a shallow, alkaline,
nutrient-rich lake that floods during the monsoon. So, pair 1 is correctly matched.
● Longwood Shola reserve forest: It is a little forest located in Tamil Nadu. It has very high species
endemicity and holds many species of trees, reptiles, frogs and birds that are endemic to the Western
Ghats. So, pair 2 is not correctly matched.
● Kanwar Lake: It is also known as Kabartal Wetland. It is Asia’s largest oxbow lake situated in Bihar. It is
a residual oxbow lake, formed due to the meandering of Gandak river, a tributary of Ganga. It is the only
Ramsar site found in Bihar. So, pair 3 is not correctly matched.
● Karaivetti Bird Sanctuary: It is one of the largest inland freshwater lakes in Tamil Nadu. It is used to
irrigate surrounding farmlands and provides foraging ground for birds migrating along the Central Asian
Flyway. So, pair 4 is correctly matched.
So, only two of the above pairs are correctly matched.
Therefore, option (b) is the correct answer.
Relevance: Longwood Shola and Karaivetti sanctuary in Tamil Nadu were recently recognised as Ramsar sites.

Q84.
Answer: a
Explanation:
● Twelve distinctive products from Arunachal Pradesh have obtained the Geographical Indication (GI)
tags. The products are Apatani, Monpa, Adi, Galo, Tai Khamti and Nyishi textiles, Monpa handmade
paper, Singpho Phalap (Singpho tea), Adi Apong, Dao (machete), Angnyat millet and Marua Apo (Marua
millet beverage).
Therefore, option (a) is the correct answer.
Knowledge Box

About Geographical Indications of Goods:


● It is defined as that aspect of industrial property which refers to the geographical
indication referring to a country or to a place situated therein as being the country or
place of origin of that product.
● Typically, such a name conveys an assurance of quality and distinctiveness which is
essentially attributable to the fact of its origin in that defined geographical locality,
region or country.
● Under the Paris Convention for the Protection of Industrial Property, geographical
indications are covered as an element of IPRs.
● They are also covered under the Trade Related Aspects of Intellectual Property Rights
(TRIPS) Agreement, which was part of the Agreements concluding the Uruguay Round
of GATT negotiations.
Relevance: Recently, 12 distinctive products of Arunachal Pradesh obtained GI tags.
45
Vajiram & Ravi PowerUp Current Affairs Test Series - 2024
CA Test No. – 10 (CA3320)
.
Q85.
Answer: b
Explanation:
● The Asia Africa Growth Corridor (AAGC) was born out of a joint declaration issued by India and Japan
in 2016. It aims for sustainable growth, with plans developed through extensive consultations in Asia
and Africa, involving diverse stakeholders. It rests on four pillars: Development and Cooperation
Projects, Quality Infrastructure and Institutional Connectivity, Building Capacities and Skills and People-
to-People partnerships. The AAGC initiative is part of the Indo-Pacific freedom corridor, jointly
established by India and Japan to counterbalance China's One Belt, One Road (OBOR) initiative. So,
statement 1 is not correct.
● India has a rich history of aiding Africa's development by offering capacity building and unique programs
such as the Pan Africa e-Network which is part of one of the pillars named Building Capacities and Skills
of the AAGC. So, statement 2 is correct.
Therefore, option (b) is the correct answer.

Q86.
Answer: d
Explanation:
• Sustainable Aviation Fuel (SAF) is a biofuel used to power aircraft that has similar properties to
conventional jet fuel but with a smaller carbon footprint. Depending on the feedstock and technologies
used to produce it, SAF can reduce life cycle GreenHouse Gas emissions dramatically compared to
conventional jet fuel. Some emerging SAF pathways even have a net-negative GHG footprint.
• Some of the sustainable feed used for producing Sustainable Aviation fuel are:
o Corn grain
o Oil seeds. So, point 1 is correct.
o Algae. So, point 3 is correct.
o Other fats, oils, and greases
o Agricultural residues
o Forestry residues
o Wood mill waste. So, point 4 is correct.
o Municipal solid waste streams
o Wet wastes (manures, wastewater treatment sludge). So, points 2 and 5 are correct.
o Dedicated energy crops.
So, all five of the above can be used for making Sustainable Aviation Fuel.
Therefore, option (d) is the correct answer.
Relevance: The Indian Institute of Petroleum is partnering with Airbus to develop sustainable aviation fuel in
India.

Q87.
Answer: a
Explanation:
● Loud Budgeting is a financial strategy characterised by the vocal and intentional allocation of funds
towards personal aspirations and valued experiences, often to friends or family. It prioritises the
allocation of resources towards personal goals and future-oriented investments that you value.

46
Vajiram & Ravi PowerUp Current Affairs Test Series - 2024
CA Test No. – 10 (CA3320)
.
● It involves a conscious decision to reject societal pressures and expectations of conspicuous
consumption in favour of personal financial values, meaningful experiences and long-term well-being.
It is all about being transparent and explicit with people in your life about what you are and aren’t willing
to spend on.
● As per experts, loud budgeting is mentally and financially healthy as it enables the individual to feel
positive without oversharing.
Therefore, option (a) is the correct answer.
Relevance: A number of stories emerged recently explaining the rise of a new viral money trend called loud
budgeting.

Q88.
Answer: c
Explanation:
● Current Account and Savings Account (CASA) is mostly used in West Asia and South-east Asia, is the
amount of money that gets deposited in the current and savings accounts of bank customers. It
indicates how much of a bank’s total deposits are in both current and savings accounts.
● Current and Savings accounts are non-term deposits and are used for daily operations. They are valid as
long as the customer wants them to be and have lower interest rates than term deposits.
● Since interest rates are lower than term deposits, CASA is a cheaper source of funds for banks. For this
reason, the CASA ratio is used to understand a bank’s financial health as it reflects the bank’s capacity
to raise money with lower borrowing costs.
Therefore, option (c) is the correct answer.
Relevance: Experts suggest that banks are unlikely to get any respite from the declining CASA ratio in the first
half of the current calendar year.

Q89.
Answer: c
Explanation:
● The International Classification of Diseases (ICD) provides a method of classifying diseases, injuries and
causes of death. The World Health Organization (WHO) publishes the ICDs to standardise the methods
of recording and tracking instances of diagnosed disease all over the world by means of codes, making
it possible to conduct research on diseases, their causes and their treatments.
● ICD-11, was adopted by the 72nd World Health Assembly in 2019 and came into effect on 1st January
2022. It is the key instrument for member countries like India to collect primary as well as secondary
data on various communicable (like malaria, TB, etc.) and non-communicable (diabetes, cancer, kidney
disease etc.) diseases and mortality statistics. So, statement 1 is correct.
● With the launch of Module-2 of the International Classification of Diseases-11 (ICD-11), Ayurveda,
Unani and Siddha (ASU) systems of medicines will now have the same morbidity codes across the
globe. It will help in furthering research related to ASU and people’s inclination towards them will rise.
So, statement 2 is correct.
Therefore, option (c) is the correct answer.
Relevance: The Prime Minister termed launch of ICD-11, Module 2 as an achievement of India.

47
Vajiram & Ravi PowerUp Current Affairs Test Series - 2024
CA Test No. – 10 (CA3320)
.
Q90.
Answer: a
Explanation:
● The Corruption Perceptions Index (CPI) is released by Transparency International. It measures how
corrupt each country’s public sector is perceived to be, according to experts and business people. A
country’s score is the perceived level of public sector corruption on a scale of 0-100, where 0 means
highly corrupt and 100 means very clean.
● The data sources used to compile the CPI specifically cover the following manifestations of public sector
corruption:
○ Bribery
○ Diversion of public funds. So, point 1 is correct.
○ Officials using their public office for private gain without facing consequences
○ Ability of governments to contain corruption in the public sector
○ Excessive red tape in the public sector which may increase opportunities for corruption
○ Nepotistic appointments in the civil service. So, point 3 is correct.
○ Laws ensuring that public officials must disclose their finances and potential conflicts of interest
○ Legal protection for people who report cases of bribery and corruption
○ State capture by narrow vested interests
○ Access to information on public affairs/government activities
● And does not cover:
○ Citizens’ direct perceptions or experience of corruption
○ Tax fraud. So, point 2 is not correct.
○ Illicit financial flows. So, point 4 is not correct.
○ Enablers of corruption (lawyers, accountants, financial advisors etc)
○ Money-laundering. So, point 5 is not correct.
○ Private sector corruption
○ Informal economies and markets
So, only two of the above are considered as parameters of corruption under the Corruption Perception Index
released by Transparency International.
Therefore, option (a) is the correct answer.
Knowledge Box

● India’s rankings slipped on the Corruption Perceptions Index (CPI) for 2023 to 93 as
against 85 it scored for 2022. India’s overall score was 39 in 2023 while it was 40 in
2022.
● However, India’s ranking is better than its neighbours Pakistan (133), Sri Lanka (115)
and Bangladesh (149) but fared low when compared to China (76).
Relevance: India’s rank on Transparency International’s Corruption Perception Index has slipped in 2024.

48
Vajiram & Ravi PowerUp Current Affairs Test Series - 2024
CA Test No. – 10 (CA3320)
.
Q91.
Answer: b
Explanation:
● E Ink displays are a special type of screen technology often used in e-readers like the Amazon Kindle.
The screens work using tiny microcapsules filled with positively charged white particles and negatively
charged black ones suspended in the fluid inside the display. So, statement 1 is correct.
● Unlike Liquid-Crystal Displays (LCD) and Light-Emitting Diode (LED) displays that use a backlight, E Ink
displays reflect light just like paper. This makes them easier on the eyes for long reading sessions. The
lack of backlighting also means that they are easier to read under brighter lighting conditions, which isn’t
the case with LCD/LED displays at all where legibility actually takes a hit under bright sunlight. So,
statement 2 is not correct.
○ E Ink displays have several key advantages which include, they consume very little power
compared to LCD and OLED displays and they cause less eye strain for the user.
● E Ink displays also have some downsides and the biggest is their slow refresh rate compared to LCD and
Organic Light-Emitting Diode (OLED) displays making them unsuitable for video or animation. E Ink also
has limitations on colour and resolution compared to other display technologies. So, statement 3 is
correct.
So, only two of the above given statements are correct.
Therefore, option (b) is the correct answer.
Relevance: There has been growing popularity of paper-like screens and e-readers like the Kindle which use E
ink displays.

Q92.
Answer: d
Explanation:
● BioRescue Project is a reproduction technology focused on saving threatened species like Northern
White Rhino. BioRescue's interdisciplinary and international scientific consortium brings together
leading experts from the various institutions of Europe, Africa and Asia for this purpose. It is funded by
the German Federal Ministry of Education and Research.
● The scientists and conservationists at BioRescue, recently have succeeded in achieving the world’s first
pregnancy of a rhinoceros after an embryo transfer.
● The successful embryo transfer and pregnancy are a proof of concept. It allows researchers to safely
move to the transfer of northern white rhino embryos.
Therefore, option (d) is the correct answer.
Knowledge Box

Northern White Rhino:


● The northern white rhinoceros is a grazing mammal with a broad, square muzzle. It
tends to be paler than rhinoceroses of other species.
● It was once abundant across Central Africa, but staggering rates of illegal hunting for its
horn have already led to its (almost certain) extinction in the wild. Now there are only
two northern white rhinoceros left, both in Kenya’s OI Pejeta Conservancy.

49
Vajiram & Ravi PowerUp Current Affairs Test Series - 2024
CA Test No. – 10 (CA3320)
.
● They are classified as Critically Endangered under The International Union for
Conservation of Nature (IUCN) Red List of Threatened Species.
Relevance: BioRescue has succeeded in achieving the world’s first pregnancy of a rhinoceros after an embryo
transfer.

Q93.
Answer: c
Explanation:
● Deep learning is a subset of machine learning that uses several layers within neural networks to do
some of the most complex machine learning tasks without any human intervention. So, statement 1 is
correct.
● Deep learning neural networks, or artificial neural networks, attempt to mimic the human brain through
a combination of data inputs, weights and bias. These elements work together to accurately recognize,
classify and describe objects within the data.
● Machine learning algorithms leverage structured, labelled data to make predictions or pre-process it
to organise into a structured format. However, a deep learning algorithm eliminates the data pre-
processing and can ingest and process unstructured data. So, statement 2 is correct.
● Deep learning has applications in many fields like law enforcement, financial services, customer service,
healthcare etc.
● Recently, scientists have reported discovering a new class of antibiotics using a form of deep learning.
In the new study, deep learning demonstrated that it can identify potential antibiotics and also explain
what substructures may contribute to antibiotic activity. It helps scientists to synthesize and test
compounds with these substrates faster.
Therefore, option (c) is the correct answer.
Relevance: Recently, scientists have reported discovering a new class of antibiotics using a form of deep
learning.

Q94.
Answer: c
Explanation:
In a new study, scientists have investigated the newly discovered class of altermagnetic materials for their
thermal properties, offering insights into the distinctive nature of altermagnets for spin-caloritronic
applications.
● Altermagnets exhibit a unique blend of magnetic characteristics, setting them apart from conventional
magnetic materials like ferromagnets and antiferromagnets.
● Properties
○ These materials exhibit properties observed in both ferromagnets and antiferromagnets,
making their study enticing. So, statement 1 is correct.
○ Altermagnets defy conventional norms by embodying a dual nature—resembling
antiferromagnets with zero net magnetization and ferromagnets with non-relativistic spin
splitting.
○ This unique behaviour emerges from the intricate interplay of atoms within the crystal structure.
50
Vajiram & Ravi PowerUp Current Affairs Test Series - 2024
CA Test No. – 10 (CA3320)
.
○ Additionally, altermagnets exhibit a unique spin polarization. The term "spin polarization"
means that a preponderance of electron spins tends to align in a particular direction. So,
statement 2 is correct.
○ The spin polarization is noteworthy in altermagnets because it occurs in the physical arrangement
of atoms (real space) and in the momentum space, where the distribution of electron spins in the
material is considered.
○ The researchers believe that altermagnets could have a pivotal role in spin caloritronics, a field
of research that explores the interplay between spin and heat flow, which are not achievable
with ferromagnets or antiferromagnets.
○ This field has potential applications in developing new technologies for information processing
and storage.
Therefore, option (c) is the correct answer.

Q95.
Answer: c
Explanation:
Codex Alimentarius Commission:
● It was jointly established by the Food and Agriculture Organisation and the World Health Organisation.
So, statement 1 is correct.
● It is an international, intergovernmental body which is based in Rome.
○ Members: It consists of 189 member countries.
○ Membership of the Commission is open to all Member Nations and Associate Members of FAO
and WHO who are interested in international food standards.
○ The Commission meets in regular sessions once a year, alternating between Geneva and Rome.
○ Funding: The programme of work of the Commission is funded through the regular budgets of
WHO and FAO, with all work subject to the approval of the two governing bodies of the parent
organisations.
○ The Commission works in the six official languages of the UN.
○ The standards of CAC are recognized by the WTO as international reference points for the
resolution of trade disputes concerning food safety and consumer protection. So, statement 3
is correct.
○ The standards developed by the Committees under the CAC, including the CCSCH, are voluntary
and the member countries of the CAC adopt and use as reference standards to align their national
standards.
○ The works of CAC contribute to the harmonization of food standards across the globe, facilitate
fair global trade in food, and enhance food safety to safeguard the health of global consumers.
So, statement 2 is correct.
So, all three of the above statements are correct.
Therefore, option (c) is the correct answer.
Relevance: The 7th session of the Codex Committee on Spices and Culinary Herbs (CCSCH) was held from 29th
January 2024 to 2nd February 2024 at Kochi.

51
Vajiram & Ravi PowerUp Current Affairs Test Series - 2024
CA Test No. – 10 (CA3320)
.
Q96.
Answer: d
Explanation:
• Metamaterials are artificially designed and manufactured novel classes of functional materials that are
designed around unique micro- and nanoscale patterns or geometry, which cause them to interact with
light and other forms of energy in ways not found in nature.
• They are also described as engineered periodic composites for altering electromagnetic properties of
materials to obtain responses that are not observed naturally.
• They are generally classified into resonant and non-resonant metamaterials. Resonant metamaterials
are those in which the meta-atoms are in the resonant state with significant changes in electromagnetic
responses; thus, producing extreme values of effective permittivity and permeability.
• Most metamaterials are anisotropic because they do not possess circular or spherical symmetry.
• Achieving a negative refractive index with metamaterials was groundbreaking because this property
was not observed in natural materials and such metamaterials enabled advancements in the fields of
optical transformations and photonics.
Therefore, option (d) is the correct answer.
Relevance: Researchers are pioneering the development of metamaterials, whose properties are not dictated
by molecular composition but by their structural geometry.

Q97.
Answer: c
Explanation:
● The International Union for Conservation of Nature (IUCN) Red List of Threatened Species, also
known as the IUCN Red List or Red Data Book, founded in 1964, is an inventory of the global
conservation status and extinction risk of biological species.
● A species is classified as critically endangered when there are fewer than 250 mature individuals. A
species is also classified as critically endangered when the number of mature individuals declines by
at least 25 per cent within three years or one generation, whichever is longer.
● The Dattatreya Night Frog is a species of amphibian in the family Nyctibatrachidae. They have sexual
reproduction. They rely on saltation to move around. They are listed as critically endangered by the
IUCN. So, point 1 is correct.
● The Malabar large-spotted civet is a viverrid endemic to the Western Ghats of India. It is listed as
Critically Endangered on the IUCN Red List. So, point 2 is correct.
● The forest owlet species is endemic to the forests of central India. These owlet species are listed as
'Endangered' by the IUCN and are on the verge of extinction. So, point 3 is not correct.
● The Namdapha flying squirrel is an arboreal, nocturnal flying squirrel endemic to Arunachal Pradesh
in northeast India. It inhabits temperate broadleaf forests. It is classified as Critically Endangered
(CR) on the IUCN Red List. So, point 4 is correct.
● The sociable lapwing species is distributed in India, Pakistan, central and southwest Asia, the Arabian
Peninsula and northeast Africa. This lapwing species is listed by IUCN as Critically Endangered. These
are monotypic species. So, point 5 is correct.
So, only four of the above species are classified as Critically Endangered in the IUCN Red List.
Therefore, option (c) is the correct answer.
Knowledge Box

52
Vajiram & Ravi PowerUp Current Affairs Test Series - 2024
CA Test No. – 10 (CA3320)
.
● Out of the nine species of mammals considered critically endangered, eight are
endemic, which means that their habitat is limited to a small geographic area
within India.
● These include the Kashmir Stag/Hangul, Malabar Large-spotted Civet andaman
Shrew, Jenkin’s Shrew, Nicobar Shrew, Namdhapa Flying Squirrel, Large Rock Rat
and Leafletted Leafnosed Bat.
● The 18 critically endangered bird species include Baer’s Pochard, Great Indian
Bustard, Sociable Lapwing, Red headed Vulture, the White Rumped Vulture, Indian
Vulture and Slender-billed Vulture.
● Of the 26 reptile species, five are endemic to India including Island Pit Viper whose
habitat is limited to a single location in the Car Nicobar Island.
● Among amphibians, several species are limited to habitats in the Western Ghats,
the Northeast and Andaman and Nicobar Islands.

Relevance: Recently, the Union Environment Ministry informed the Rajya Sabha citing a report of the
International Union for Conservation of Nature (IUCN) that seventy-three species in India are critically
endangered.

Q98.
Answer: d
Explanation:
● Smart Rings pack impressive technology into a small ring-sized device. They can monitor health stats,
control other devices, and much more, much like their bulkier smartwatch counterparts.
● The Oura Ring is perhaps the most popular example that lets one keep tabs on their health and wellness.
There’s also the Happy Ring, which goes a step further by offering personalised breathing exercises and
guided meditations to help users improve their mood. So, point 1 is correct.
● Another convenient feature is contactless payment. Rings like the McLear RingPay let one link their bank
account or card to the ring. Then, when one is shopping at a store with a compatible point-of-sale system,
one can simply hover their hand over the terminal to complete a payment. So, point 2 is correct.
● Smart home control is another popular use case. Smart rings typically include gesture recognition, so
one can perform gestures like twisting or tapping the ring to do things like turn off lights, adjust your
thermostat, unlock doors, and more. So, point 3 is correct.
● An up-and-coming online security feature in smart rings is biometric authentication, which provides a
more secure way to navigate around the web than passwords and PINs. Motiv is doing a beta test for
fingerprint scanning and facial recognition as authentication methods. So, point 4 is correct.
So, at the present level of technology, all four of the above activities can be successfully carried out by using
smart rings.
Therefore, option (d) is the correct answer.
Relevance: Often billed as the future of wearables, smart rings are perhaps one of the most discreet gadgets
around that effortlessly blend technology with fashion.

53
Vajiram & Ravi PowerUp Current Affairs Test Series - 2024
CA Test No. – 10 (CA3320)
.
Q99.
Answer: c
Explanation:
● Cyclocenes are a new class of organometallic compounds characterized by repeating units of sandwich
complexes which are commonly used as building blocks in this branch of chemistry. They are synthesized
by Scientists from the Karlsruhe Institute of Technology (KIT) and the University of Marburg.
● The discovery of these cyclic nanometre-scale sandwich complexes represents a significant advancement
in organometallic chemistry. With the ability to form closed rings from individual building blocks,
researchers have expanded their toolbox for creating new functional organometallic materials.
● These compounds deviate significantly from the erstwhile linear structure of sandwich compounds. They
consist of 18 repeating metallocene sandwich units, forming almost ideally circular, closed rings in the
solid state.
● It is a nano ring with an outer diameter of 3.8 nanometers and can exhibit orange-coloured
photoluminescence. The emergence of cyclocene broadens the scope of research into different ring sizes
and unique physical properties, thereby augmenting the organometallic chemistry toolkit.
Therefore, option (c) is the correct answer.
Relevance: Scientists from the Karlsruhe Institute of Technology (KIT) and the University of Marburg have
synthesized cyclocenes from sandwich compounds.

Q100.
Answer: a
Explanation:
● Maximum four nomination papers can be filed by a candidate or on his behalf for a constituency. If a
candidate desires to contest from a constituency other than the constituency in which he has been
enrolled as an elector, he is required to produce a copy of the electoral roll or a relevant part of the
electoral roll in which he has been registered as an elector. So, statement 1 is not correct.
● As per Articles 84 & 173, a candidate is required to make and subscribe to an oath in the form
prescribed under the third schedule of the Constitution to the effect that he has true faith and integrity
in the Constitution and he supports sovereignty and unity of India. Such an oath is required to be made
after filing the nomination and, in any case, before the date of scrutiny. That means such an oath must
be subscribed after filing the nomination and on the day before the date of scrutiny.
● As per the Representation of the People Act, 1951, a candidate is required to make a security deposit
prescribed by law. At present, for contesting an election for Lok Sabha, the sum required to be deposited
by a candidate is Rs. 25,000/-. For contesting an Assembly Election, the sum required to be deposited by
a candidate is Rs. 10,000/-. In addition to this, even if a candidate has filed four nomination papers
from the same constituency, he is required to make a deposit only once. The security deposit can be
deposited in cash, with the Returning Officer and by chalan in the Reserve Bank of India or treasury. So,
statement 2 is not correct.
● Candidate is required to sign his nomination paper after making several declarations. Out of these
declarations, the candidate is required to make a declaration as to on which symbol he wants to
contest the election. Before filing nomination, the candidate is required to mention his choice of
symbols. So, statement 3 is correct.
54
Vajiram & Ravi PowerUp Current Affairs Test Series - 2024
CA Test No. – 10 (CA3320)
.
So, only one of the above statements is correct.
Therefore, option (a) is the correct answer.
Relevance: The Delhi State Election Commission rejected 45% of the nominations filed by the 15 political parties
and unattached aspirants.

55
Vajiram & Ravi PowerUp Current Affairs Test Series - 2024
CA Test No. – 10 (CA3320)

You might also like